SlideShare a Scribd company logo
1 of 20
1. A patient is followed up in an                    revealed localized bright-red hyperemia;
endocrinological dispensary on account of            tonsils are swollen, soft, lacunas contain pus,
hyperthyreosis. Weight loss, tachycardia, finger     tongue is crimson. Cervical lymph nodes are
tremor are accompanied by hypoxia                    enlarged, dense and painful. What is the
symptoms - headache, fatigue, eye flicker.           most probable diagnosis?
What mechanism of thyroid hormones action            A. Rubella
underlies the development of hypoxia?                B. Scarlet fever
A. Intensification of respiratory ferment            C. Infectious mononucleosis
synthesis                                            D. Whooping cough
B. Inhibition of respiratory ferment synthesis       E. Diphtheria
C. Specific binding of active centres of             6. A 45 y.o. woman suffers from Cushing's
respiratory                                          syndrome - steroid diabetes. Biochemical
ferments                                             examination revealed: hyperglycemia,
D. Disjunction, oxydation and phosphorilation        hypochloremia. Which of the undermentioned
E. Competitive inhibition of respiratory             processes is the first to be activated?
ferments                                             A. Gluconeogenesis
2. A laboratory received a material from             B. Glycolysis
a patient's wound. Ppreliminary diagnosis            C. Glycogenolysis
is gaseous gangrene. What microbiological            D. Glucose transport to the cell
method should be applied to determine                E. Glucose reabsorption
species of causative agent?                          7. Examination of an isolated cardiomyocyte
A. Bacteriological                                   revealed that it didn't generate excitation
B. Allergic                                          impulses automatically. This cardiomyocyte
C. Serological                                       was obtained from:
D.RIA                                                A. Sinoatrial node
E. Bacterioscopic                                    B. Purkinje's fibers
3: A 4 y.o. child with signs of durative proteinic   C. Ventricles
starvation was admitted to the hospital.             D. Atrioventricular node
The signs were as follows: growth inhibition,        E. His' bundle
anemia, edemata, mental deficiency Choose            8. Inhabitants of territories with cold climate
a cause of edemata development: .                    have high content of an adaptive
A. Reduced synthesis of albumins                     thermoregulatory hormone. What hormone
B. Reduced synthesis of hemoglobin                   is meant? .
C. Reduced synthesis of glycoproteins                A. Insulin
D. Reduced synthesis of globulins                    B. Glucagon
E. Reduced synthesis of lipoproteins                 C. Throxin
4. A 45 y.o. patient consulted a doctor              D. Cortisol
about plaque-shaped formation on his neck.           E. Somatotropin
Histological examination of biopsy skin              9. As a result of damage to certain structures
material revealed tumourous cells of round           of brainstem an animal lost orientation
and oval form with thin ring of basophilic           reflexes. What structures were damaged?
cytoplasma that resemble of cells of basal           A. Medial nuclei of reticular formation
epidermal layer. What tumour is it?                  B. Quadritubercular bodies
A. Hydradenoma                                       C. Black substance
B. Epidermal cancer                                  D. Reel nuclei
C. Trichoepithelioma                                 E. Vestibular nuclei
D. Basalioma                                         10. According to the data of WHO, for about
E. Syringoadenoma                                    250 mln of Earth population fall ill with
5. A boy is 7 y.o. Objectively: against the          malaria. This disease is mostly spread in
background of hyperemic skin there is                tropical and subtropical regions. Range of
knobby bright-pink rash on his forehead,             its spread falls into the areal of the following
neck, at the bottom of abdomen, in the               mosquitoes:
popliteal spaces; nasolabial triangle is pale.       A. Aedes
Examination of oropharyngeal surface                 B. Mansonia
C. Culex                                              16. A patient was diagnosed with active
D. Culiseta                                           focal pulmonary tuberculosis. What drug
E. Anopheles                                          should be prescribed in the first place?
11. A 35 y.o. patient who often consumes alcohol      A. Ethionamide
was treated with diuretics. There                     B. Isoniazid
appeared serious muscle and heart                     C. Ethoxide
weakness, vomiting, diarrhea, AP- 100/60              D. Sulfalen
mm Hg, depression. This condition is caused           E. Cyclocerine
by intensified excretion with urine of:               17. Examination of a young man in the AIDS
A. Potassium-                                         centre produced a positive result of
B. Phosphates                                         immune-enzyme assay with HIV antigens.
C. Chlorine                                           Patient's complaints about state of his health
D. Sodium                                             were absent. What can the positive result of
E. Calcium                                            immune-enzyme assay be evidence of?
12. On the 6th day of treatment a patient             A. HIV infection
with acute renal insufficiency developed              B. Being infected with HBV
polyuria. Diuresis intensification at the beginning   C. Having had AIDS recently
of polyuria stage of acute renal insufficiency        D. Being ill with AIDS
is caused by:                                         E. HBV persistence
A. Growth of natriuretic factor                       18. RNA that contains AIDS virus
B. Reduction of aldosteron content in plasma          penetrated into a leukocyte and by means
C. Reduction of vasopressin content in                of reverse transcriptase forced a cell to
plasma                                                synthetize a viral DNA. This process is
D. Renewal of filtration in nephrons                  based upon:
E. Volume expansion of circulating blood              A. Operon repression
13. Osmotic pressure of a man's blood                 B. Operon depression
plasma is 350 mosmole/l (standard pressure            C. Reverse transcription
is 300 mosmole/l). First of all it will result in     D. Convariant replication
high secretion of the following hormone:              E. Reverse translation
A. Vasopressin                                        19. A patient is ill with diabetes mellitus that
B. Adrenocorticotropin                                is accompanied by hyperglycemia of over 7,2
C. Cortisol                                           millimole/l on an empty stomach. The level
D. Natriuretic                                        of what blood plasma protein allows to estimate
E. Aldosteron                                         the glycemia rate retrospectively (4-8
14. Neurological examination of a 65 y.o.             weeks before examination)?
patient revealed a haemorrhage within the             A. Ceruloplasmin
superior temporal gyrus. In the blood supply          B. Glycated hemoglobin
area of which artery is it?                           C. Fibrinogen
A. Middle cerebral artery                             D. Albumin
B. Anterior communicating artery                      E. C-reactive protein
C. Anterior cerebral artery                           20. A sportsman was recommended to take
D. Basilar artery                                     a medication that contains carnitine in order
E. Posterior cerebral artery                          to improve his results. What process is activated
15. A 32 y.o. man is tall, he has gynecomastia,       by carnitine the most?
adult woman pattern of hair distribution, high        A. Synthesis of ketone bodies
voice, mental deficiency, sterility. Provisional      B. Synthesis of steroid hormones
diagnosis is Klinefelter's syndrome. In               C. Tissue respiration
order to specify diagnosis it is necessary to         D. Fatty acids transport to mitochondrions
analize:                                              E. Synyhesis of lipids
A. Leukogram                                          21. A patient consulted a doctor about
B. Genealogy                                          bowels disfunction. The doctor established
C. Blood group                                        symptoms of duodenitis and enteritis.
D. Spermatogenesis                                    Laboratory examination helped to make the
E. Caryotype                                          following diagnosis: lambliosis. What medication
should be administered?                         B. Hyaline degeneration
A. Tetracycline                                 C. Carbohydrate degeneration
B. Monomycin                                    D. Fatty vascular-stromal degeneration
C. Metronidazole                                E. Amyloidosis
D. Erythromycin                                 27. Labeled aminoacids alanine and
E. Chingamin                                    tryptophane were introducted to a mouse
22. A man took a quiet expiration. Name an      in order to study localization of protein
air volume that is meanwhile contained in       biosynthesis in its cells. Around what
his lungs:                                      organellas will the accumulation of labeled
A. Expiratory reserve volume                    aminoacids be observed?
B. Functional residual capacity                 A. Ribosomes
C. Residual volume                              B. Golgi apparatus
D. Vital lung capacity                          C. Agranular endoplasmic reticulum
E. Respiratory volume                           D. Cell centre
23. During pubescence the cells of male         E. Lysosomes
sexual glands begin to produce male             28. 48 hours after performing tuberculin
sex hormon testosterone that calls forth        test (Mantoux test) to a child a 10
secondary sexual characters. What cells of      mm papule appeared on the spot of
male sexual glands produce this hormone?        tuberculin introduction. What hypersensitivity
A. Sertoli's cells                              mechanism underlies these changes?
B. Leidig cells                                 A. Antibody-dependent cytotoxicity
C. Sustentocytes                                B. Granulomatosis
D. Spermatozoa                                  C. Cellular cytotoxicity
E. Supporting cells                             D. Immune complex cytotoxicity
24. A 16 y.o. boy from a countryside entered    E. Anaphylaxis
an educational establishment. Scheduled         29. A patient who has been suffering from
Manteux test revealed that the boy had          cardiac insufficiency for several months
negative reaction. What are the most            has been taking digoxin on an outpatient
reasonable actions in this case?                basis. At a certain stage of treatment there
A. To isolate the boy temporarily from his      appeared symptoms of drug overdose. What
mates                                           phenomenon underlies the development of
B. To perform serodiagnostics of tuberculosis   this complication?
C. To perform BCG vaccination                   A. Functional cumulation
D. To perform rapid Price diagnostics           B. Tachyphylaxis
E. To repeat the reaction in a month            C. Habituation
25. Autopsy of a 56 y.o. man revealed in the    D. Sensibilization
right temporal part of brain a big focus of     E. Material cumulation
softened grey matter that was semi-liquid       30. Examination of a patient revealed
and light grey. Arteries of cerebral tela       reduced contents of magnesium ions that
contain multiple whitish-yellow thiekenings     are necessary for attachment of ribosomes
of intima that abruptly narrow the lumen.       to the granular endoplasmatic reticulum.
What is your diagnosis?                         It is known that it causes disturbance of
A. Brain edema                                  protein biosynthesis. What stage of protein
B. Brain abscess                                biosynthesis will be disturbed?
C. Hemorrhage                                   A. Aminoacid activation
D. Hemorrhagic infarction                       B. Translation
E. Ischemic stroke                              C. Termination
26. Autopsy of a man who died from              D. Replication
chronic cardiovascular collapse revealed        E. Transcription
"tiger heart". Sidewards of endocardium         31. A child complains of general weakness,
a yellowish-white banding can be seen;          loss of appetite, a troubled sleep, itching in
myocardium is dull, dark-yellow. What           the perianal area. The provisional diagnosis
process caused this pathology?                  is enterobiasis. In order to specify this diagnosis
A. Fatty parenchymatous degeneration            it is necessary to perform:
A. Roentgenoscopy                                 C. Necrotic
B. Duodenal contents analysis                     D. Fibrinous
C. Scraping from perianal folds                   E. Catarrhal
D. Immune diagnostics                             37. A 70 y.o. man has cut an abscess off
E. Biopsy of muscle tissue                        in the area of mammiform process during
32. In some regions of South Africa there is      shaving. Two days later he was admitted
a spread sickle-shaped cell anemia, in which      to the hospital with inflammation of
erythrocytes have shape of a sickle as a          arachnoid membranes. How did the infection
result of substitution of glutamin by valine in   penetrate into the cavity of skull?
the hemoglobin molecule. What is the cause        A. V,v.auriculares
of this disease?                                  B. V.v.labyrinthi
A. Genomic mutations                              C. V.facialis
B. Crossingover                                   D. V.v.tympanicae
C. Gene mutation                                  E. V.emissariae mastoideae
D. Transduction                                   38. 24 hours after appendectomy blood of a
E. Disturbance of mechanisms of genetic           patient presents neutrophilic leukocytosis
information realization                           with regenerative shift. What is the
33. A patient suffers from vision impairment      most probable mechanism of leukocytosis
- hemeralopy (night blindness). What vitamin      development?
preparation should be administered                A. Amplification of leukopoiesis
the patient in order to restore his vision?       B. Decelerated leukocyte destruction
A. Thiamine chloride                              C. Redistribution of leukocytes in the organism
B. Tocopherol acetate                             D. Deceleratied emigration of leukocytes to
C. Retinol acetate                                the tissues
D. Vicasol                                        E. Amplification of leukopoiesis and
E. Pyridoxine                                     decelerated emigration of leukocytes to
34. A patient presents high activity of           the tissues
LDH1.2 , aspartate aminotransferase, creatine     39. Examination of cell culture got from a
phosphokinase. In what organ (organs) is          patient with lysosomal pathology revealed
the development of a pathological process         accumulation of great quantity of lipids in
the most probable?                                the lysosomes. What of the following diseases
A. In the heart muscle (initial stage of          is this disturbance typical for?
myocardium infarction)                            A. Phenylketonuria
B. In skeletal muscles (dystrophy, atrophy)       B. Galactosemia
C. In kidneys and adrenals                        C. Gout
D. In liver and kidneys                           D. Tay-Sachs disease
E. In connective tissue                           E. Wilson disease
35. A woman who has been keeping to a             40. A group of mountain climbers went
clean-rice diet for a long time was diagnosed     through the blood analysis at the height of
with polyneuritis (beriberi). What vitamin        3000 m. It revealed decrease of HCO3 to 15
deficit results in development of this disease?   micromole/l (standard is 22-26 micromole/l).
A. Thiamine                                       What is the mechanism of HCO3 decrease?
B. Pyridoxine                                     A. Hypoventilation
C. Ascorbic acid                                  B. Intensification of acidogenesis
D. Folic acid                                     C. Hyperventilation
E. Riboflavin                                     D. Decrease of bicarbonate reabsorption in
36. Colonoscopy of a patient ill with             kidneys
dysentery revealed that mucous membrane           E. Decrease of ammoniogenesis
of his large intestine is hyperemic, edematic,    41. A patient ill with diabetes mellitus felt
its surface was covered with grey-and-green       acute pain in his right foot. Objectively: foot
coats. Name the morphological form of             thumb is black, foot tissues are edematous.
dysenteric collitis:                              there are foci of epidermis desquamation,
A. Ulcerous                                       stinking discharges. What clinicopathological
B. Purulent                                       form of necrosis is it?
A. Infarction                                   C. Decrease of ESR
B. Dry gangrene                                  D. Hypercoagulation
C. Moist gangrene                                E. Increase of ESR
D. Bedsore                                       47. Histological examination of a 40 y.o.
E. Sequestrum                                    man's thymus revealed decreased share of
42. Power inputs of a boy increased from 500     parenchymatous gland elements, increased
to 2000 kJ pro hour. What can be the cause       share of adipose and loose connective tissue,
of it?                                           its enrichment with thymus bodies.
A. Food intake                                   The organ's mass was unchanged. What
B. Raise of outer temperatute                    phenomenon is it?
C. Mental activity                               A. Accidental involution
D. Transition from sleep to wakefulness          B. Atrophy
E. Physical exercise                             C. Dystrophy
43. For the preparation of a patient`s           D. Hypotrophy
burn skin surface a certain medication was       E. Age involution
used. Its antiseptic action is provided by       48. A patient who suffers from cancer of
free oxygen that segregates in presence          back of tongue has an intense bleeding as a
of organic substances. Choose the right          result of affection of dorsal lingual artery by
answer:                                          the tumour. What vessel should be ligated to
A. Potassium permanganate                        stop bleeding?
B. Chlorhexidine                                 A. Facial artery
C. Sodium bicarbonate                            B. Dorsal lingual artery
D. Boric acid                                    C. Ascending pharyngeal artery
E. Furacilin                                     D. Lingual artery
44. Bacteriological examination of a patient     E. Deep lingual artery
with food poisoning required inoculation of      49. In course of indirect histogenesis of
a pure culture of bacteria with the following    tubular bone tissue a plate is formed
properties: gram-negative movable bacillus       between epiphyseal and diaphyseal ossification
that grows in the Endo's medium in form of       centres that provides further
colourless colonies. A representative of which   lengthwise growth of bones. What structure
species caused this disease?                     is it?
A. Esherichia                                    A. Osseous plate
B. Salmonella                                    B. Metaphyseal plate
C. Shigella                                      C. Osteon
D. Citrobacter                                   D. Osseous cuff
E. Iersinia                                      E. Layer of interior general plates
45. A lymph node punctate of a patient with      50. A man with cut wound of his right foot
suspected protozoal disease was examined.        sole was admitted to the hospital ward. The
Examination of the stained specimen              patient has limited elevation of the lateral
(Romanovsky's stain) revealed some               foot edge. In course of wound management
crescent bodies with pointed end, blue           the injury of a muscle tendon was revealed.
cytoplasm and red nucleus. What protozoan        What muscle is injured?
were revealed in the smears?                     A. Triceps muscle of crus
A. Viscerotropic leishmania                      B. Long extensor muscle of toes
B. Toxoplasms                                    C. Long peroneal
C. Trypanosomes                                  D. Short peroneal
D. Malarial plasmodiums                          E. Anterior tibial
E. Demtropic leishmania                          51. After resection of the middle third of
46. Long-term starvation cure of a patient       femoral artery obliterated by a thromb the
resulted in diminished ratio of albumines        lower extremity is supplied with blood due
and globulines in plasma. What of the            to the surgical bypass. Name an artery that
following will be result of these changes?       plays the main role in reestablishment of
A. Decrease of hematocrit                        blood flow:
B. Increase of hematocrit                        A. Deep femoral artery
B. Deep external pudendal artery                    branches of carotid artery. For a temporary arrest
C. Descending genicular artery                      of bleeding it is necessary to press the
D. Superficial circumflex artery of hip bone        carotid artery to the tubercle of a cervical
E. Superficial epigastric artery                    vertebra. Which vertebra is it?
52. An experimental animal was first sensibilized   A.VI
whereupon an antigen dose was                       B. II
introduced subcutaneously. This injection           C. IV
resulted in the development of a fibrinous          D. III
inflammation with alteration of vessel              E. V
walls, basal substance and fibrous structures       57. Reaction of passive hemagglutination
of connective tissue in form of mucoid              conducted with erythrocytic typhoid
and fibrinoid swelling and necrosis. What           Vidiagnosticum
immunological reaction took place?                  helped to reveal some antibodies
A. Immediate hypersensitivity                       in the dilution of the patient's serum
B. Granulomatosis                                   at a ratio of 1:80 that exceeds the diagnostic
C. Reaction of transplantation immunity             titer. Such result witnesses of:
D. Delayed-type hypersensitivity                    A. Typhoid fever recurrence
E. Normergic reaction                               B. Being a potential carrier of typhoid bacilli
53. Autopsy of a patient who suffered               C. Being ill with acute typhoid fever
from croupous pneumonia and died from               D. Incubation period of typhoid fever
pneumococcal sepsis revealed 900 ml of              E. Reconvalescence of a patient ill with
turbid greenish-yellow liquid in the right          typhoid fever
pleural cavity. Pleural leaves are dull,            58. A patient with clinical presentations
plephoric. Name the clinicopathological             of immunodeficiency went through
form of inflammation in the pleural cavity:         immunological examinations. They revealed
A. Acute abscess                                    significant loss of cells that form rosettes with
B. Empyema                                          erythrocytes of a ram. What conclusion
C. Phlegmon                                         can be made according to the analysis data?
D. Fibrinous inflammation                           A. Decrease of natural killer cell rate
E. Chronic abscess                                  B. Decrease of T-lymphocytes rate
54. A patient was delivered to the hospital         C. Insufficiency of effector cells of humoral
by an emergency team. Objectively:                  immunity
grave condition, unconscious, adynamy.              D. Decrease of B-lymphocytes rate
Cutaneous surfaces are dry, eyes are sunken,        E. Decrease of complement system rate
face is cyanotic. There is tachycardia and          59. Examination of a 60 y.o. patient revealed
smell of acetone from the mouth. Analysis           hyperglycemia and glucosuria. A doctor
results: blood glucose - 20,1 micromole/l           administered him a medication for internal
(standard is 3,3-5,5 micromole/l), urine            use. What medication is it?
glucose - 3,5% (standard is - 0). What is the       A. Oxytocin
most probable diagnosis?                            B. Corglycon
A. Hypoglycemic coma                                C. Furosemide
B. Acute heart failure                              D. Pancreatine
C. Anaphylactic shock                               E. Glibenclamid
D. Hyperglycemic coma                               60. In order to estimate toxigenity of diphtheria
E. Acute alcoholic intoxication                     agents obtained from patients the
55. A patient has a disturbed absorbtion of         cultures were inoculated on Petri dish with
fat hydrolysates. It might have been caused         nutrient agar on either side of a filter paper
by a deficit in the small intestine cavity:         strip that was put into the centre and moistened
A. Of bile pigments                                 with antidiphtherie antitoxic serum.
B. Of sodium ions                                   After incubation of inoculations in agar the
C. Of bile acids                                    strip-like areas of medium turbidity were
D. Of lipolytic enzymes                             found between separate cultures and the
E. Of liposoluble vitamins                          strip of filter paper. What immunological
56. An injured man has bleeding from                reaction was conducted?
A. Opsonization reaction                          E. 70
B. Coomb's test                                   66. A 40 y.o. patient complains of intensive
C. Agglutination reaction                         heartbeats, sweating, nausea, vision impairment,
D. Rings precipitation reaction                   arm tremor, hypertension. From his
E. Precipitation gel reaction                     anamnesis: 2 years ago he was diagnosed with
61. A woman with III (B), Rhˉ blood group         pheochromocytoma. Hyperprocduction of
born a child with II (A) blood group. The         what hormones causes the given pathology?
child is diagnosed with hemolytic disease of      A. ACTH
newborn as a result of rhesus incompatibility.    B. Thyroid hormones
What blood group is the child's father            C. Aldosterone
likely to have?                                   D. Glucocorticoids
A. I (0). Rh +                                    E. Catecholamines
B. III (B). Rh+                                   67.Mucous membrane of the right palatine tonsil
C. I (0), Rh-                                     has a painless ulcer with smooth lacquer fundus
D. II (A), Rh-                                    and regular cartilaginous edges. Microscopically:
E. II (A), Rh+                                    inflammatory infiltration that consists of
62. A 12 y.o. boy who suffers from bronchial      lymphocytes, plasmocytes, a small number of
asthma has an acute attack of asthma: evident     neutrophils and epithelioid cells; endovasculitis
expiratory dyspnea, skin pallor. What             and perivasculitis. What disease is it?
type of alveolar ventilation disturbance is it?   A. Actinomycosis
A. Central                                        B. Pharyngeal diphtheria
B. Neuromuscular                                  C. Ulcerous necrotic Vincent’s angina
C. Throracodiaphragmatic                          D. Tuberculosis
D. Restrictive                                    E. Syphilis
E. Obstructive                                    68. For the purpose of retrospective diagnostics of
63. A patient diagnosed with carcinoid            recent bacterial dysentery it was decided to
of bowels, was admitted to the hospital.          perform serological examination of blood serum
Analysis revealed high production of              in order to determine antibody titer towards Shiga
serotonin. It is known that this substance        bacilli. What of the following reactions should be
is formed of tryptophane aminoacid.               applied?
What biochemical mechanism underlies this         A. Passive hemagglutination
process?                                          B. Bordet-Gengou test
A. Microsomal oxydation                           C.Hemolysis
B. Formation of paired compounds                  D. Precipitation
C. Desamination                                   E. Bacteriolysis
D. Decarboxylation                                69. Autopsy of a 48 y.o. man revealed a round
E. Transamination                                 formation 5 cm in diameter with clear-cut
64. Examination of a patient revealed an          outlines in the region of the 1st segment of his
abscess of pterygopalatine fossa. Where can       right lung. This formation was encircled with a
the infection spread to unless the disease is     thin layer of connective tissue full of white brittle
managed in time?                                  masses. Make a diagnosis of the secondary
A. To the orbit                                   tuberculosis form:
B. To the subgaleal temporal space                A. Fibrous cavernous tuberculosis
C. To the tympanic cavity                         B. Tuberculoma
D. To the interpterygoid space                    C. Acute focal tuberculosis
E. To the frontal sinus                           D. Caseous pneumonia.
65. Examination of a man established that         E. Acute cavernous tuberculosis
cardiac output equaled 3500 ml, systolic          70. A man’s intrapleural pressure is being
output - 50 ml. What is the man's heart rate      measured. In what phase did the man hold his
pro minute?                                       breath, if his pressure is 7,5 cm Hg?
A.60                                              A. Forced expiration
B.90                                              B. Quiet inspiration
C. 50                                             C. –
D. 80                                             D. Quiet expiration
E. Forced inspiration                                  B. Fibrinous inflammation
71. In the surgical department of a hospital there     C. Hyalinosis
was an outbreak of hospital infection that showed      D. Mucoid swelling
itself in often postoperative wound abscesses.         E. Amyloidosis
Bacteriological examination on of pus revealed         76. In course of an operation surgeon
aurococcus. What examination shall be conducted        removed a part of a lung that was ventilated
to find out the source of this causative agent         by a tertiary bronchus accompanied
among the department personnel?                        by branches of pulmonary artery and other
A. Serological identification                          vessels. What part of a lung was removed?
B. Microscopical examination                           A. Middle lobe
C. Phagotyping                                         B. Bronchopulmonary segment
D. Biochemical identification                          C. Superior lobe
E. Estimation of antibiotic susceptibility             D. Inferior lobe
72. Heart rate of a man permanently                    E. Pulmonary lobule
equals 40 beats pro minute. What is the                77. Vitamin A deficit results in the impairment
pacemaker?                                             of twilight vision. Name the cells that
A. His' bundle                                         have the above-mentioned photoreceptor
B. Purkinje's fibers                                   function:
C. His' bundle branches                                A. Ganglion neurocytes
D. Sinoatrial node                                     B. Rod receptor cell
E. Atriventricular node                                C. Cone receptor cells
73. A 4 y.o. boy has had recently serious              D. Horizontal neurocytes
viral hepatitis, Now there are such                    E. Bipolar neurons
clinical presentations as vomiting, loss of            78. Microscopical examination of a removed
consciousness, convulsions. Blood analysis             appendix revealed an edema, diffuse
revealed hyperammoniemia. Disturbunce                  neutrophilic infiltration of appendix wall
of which biochemical process caused such               along with necrosis and defect of mucous
pathological condition of the patient?                 membrane with affection of its muscle plate.
A. Disturbed neutralization of ammonia in              What appendicitis form was developed?
liver                                                  A. Gangrenous
B. Inhibition of transamination enzyms                 B. Ulcerophlegmonous
C. Increased putrefaction of proteins in               C. Apostematous
bowels                                                 D. Superficial
D. Activation of aminoacid decarboxylation             E. Phlegmonous
E. Disturbed neutralization of biogenic                79. Introduction of a pharmaceutical
amines                                                 substance to an experimental animal
74. An experimental rat with extremity                 resulted in reduction of salivation, pupil
paralysis has no tendon and cutaneous                  mydriasis. Next intravenous introduction of
reflexes, muscle tone is decreased, but                acetylcholine didn't lead to any significant
muscles of the affected extremity maintain             changes of heart rate. Name this substance:
their ability to react with excitation to the direct   A. Proserin
action of continious current. What type                B. Atropine
of paralysis is it?                                    C. Propranolol
A. Spastic peripheral                                  D. Adrenaline
B. Spastic central                                     E. Salbutamol
C. Extrapyramidal                                      80. Examination of a 43 y.o, anephric patient
D. Flaccid peripheral                                  revealed anemia symptoms. What is the
E. Flaccid central                                     cause of these symptoms?
75. Autopsy of a 58 y.o, man revealed that             A. Folic acid deficit
bicuspid valve was deformed, thickened and             B. Iron deficit
unclosed. Microscopically: foci of collagen            C. Reduced synthesis of erythropoietins
fibrilla are eosinophilic, react positively to         D. Vitamin B12 deficit
fibrin. The most probably it is:                       E. Enhanced destruction of erythrocytes
A. Fibrinoid swelling                                  81. Examination of a newborn boy's genitals
revealed a cleft of urethra that opens             retrosternal pain a patient presented a jump
on the inferior surface of his penis. What         of aspartate aminotransferase activity in
developmental anomaly is it?                       blood serum. What pathology is this deviation
A. Cryptorchism                                    typical for?
B. Epispadia                                       A. Myocardium infarction
C. Monorchism                                      B. Viral hepatitis
D. Hennaphroditism                                 C. Diabetes mellitus
E. Hypospadia                                      D. Collagenosis
82. A patient ill with collagenesis has            E. Diabetes insipidus
been taking prednisolone for a long time.          87. A peripheral segment of vagus nerve on
Hypokaliemia development caused spastic pain of    a dog's neck was being stimulated in course
skeletal muscles. What medication                  of an experiment. The following changes
should be used in order to correct potassium       of cardiac activity could be meanwhile
exchange?                                          observed:
A. Dithylinum                                      A. Heart rate and heart force amplification
B. Panallgin                                       B. Enhancement of atrioventricular
C. Thyrocalcitonin                                 conduction
D. Diazepam                                        C. Heart rate fall
E. Noshpa                                          D. Heart hurry
83. From pharynx of a child with suspected         E. Increased excitability of myocardium
diphtheria a pure culture of microorganisms        88. A 39 y.o. woman went through an
was isolated. Their morphological tinctorial,      operation in course of which surgeons
cultural and biochemical properties                removed her uterine tube that was enlarged
appeared to be typical for diphtheria              and a part of an ovary with a big cyst.
causative agents. What study should be             Histological examination of a tube wall
conducted in order to drow a conclusion that       revealed decidual cells, chorion villi. What
this is a pathogenic diphtheria bacillus?          was the most probable diagnosis made after
A. Estimation of proteolytic properties            examination of the uterine tube?
B. Estimation of urease activity                   A. Tubal pregnancy
C. Estimation of cystinous activity                B. Placental polyp
D. Estimation of ability to decompose starch       C. Papyraceous fetus
E. Estimation of toxigenic properties              D. Lithopedion
84. A man was admitted to the hospital             E. Choriocarcinoma
on the 5th day of disease that manifested          89. A forensic medical expert examines
itself by jaundice, muscle aching, chill, nose     the body of a 58 y.o. man who had been
bleedings. In course of laboratory diagnostics     consuming large amounts of alcochol for a
a bacteriologist performed dark-field microscopy   long time and died at home. Microscopicaly:
of the patient's blood drop. Name                  the right lung is dense and enlarged, its incision
a causative agent of this disease:                 revealed that the tissue is greyish and
A. Leptospira interrogans                          homogenous, pleura is covered with greyish
B. Bartinella bacilloformis                        layers. Microscopically - alveolar cavities
C. Calymmatobacterium granulomatis                 contain fibrin, hemolyzed erythrocytes.
D. Rickettsia mooseri                              Make a diagnosis:
E. Borrelia dutlonii                               A. Focal pneumonia
85. Nappies of a newborn have dark spots           B. Caseous pneumonia
that witness of formation of homogentisic          C. Interstitial pneumonia
acid. Metabolic imbalance of which                 D. Primary pulmonary tuberculosis
substance is it connected with?                    E. Croupous pneumonia
A. Methionine                                      90. A 40 y.o. woman was admitted to the
B. Thyrosine                                       infectious diseases department with high
C. Galactose                                       body temperature. Objectively: evident
D. Tryptophane                                     meningeal symptoms. A spinal cord punction
E. Cholesterine                                    was made. What anatomic formation was
86. 12 hours after an accute attack of             punctured?
A. Cisterna cerebellomedullaris posterior          tissue will be inhibited?
B. Spatium subdurale                               A. Of fibroblasts
C. Spatium epidurale                               B. Of pericytes
D. Spatium subarachnoideum                         C. Of pigment cells
E. Cavum trigeminale                               D. Of lipocytes
91. A patient with II stage hypertension has       E. Of macrophags
been taking one of hypotensive medications         96. 6 months after delivery a woman had
for the purpose of treatment. After a time         uterine bleeding. Gynecological examination
arterial pressure decreased, but the patient       revealed in the uterine cavity a dark-red
started complaining of flaccidity, sleepiness,     tissue with multiple cavities that resembled
indifference. A bit later he felt stomach          of "sponge". Microscopic examination of
pain. He was diagnosed with ulcer. What            the tumour revealed some atypic light epithelial
hypotensive medication has the patient been        Langhans cells and giant cells of
taking?                                            cyncytiotrophoblast in blood lacunas. What
A. Captopril                                       tumour is it?
B. Dibazoie                                        A. Vesicular mole
C. Reserpine                                       B. Fibromyoma
D. Furosemide                                      C. Adenocarcinoma
E. Verapamil                                       D. Squamous cell nonkcratinous carcinoma
92. A patient had to go through an operation.      E. Chorioepithelioma
Doctors introduced him dithylinum                  97. A 50 y.o. patient with chronic cardiac
(listenone) and performed intubation.              insufficiency and tachyarrythmia was
After the end of operation and cessation           prescribed a cardiotonic drug. What drug
of anesthesia the independent respiration wasn't   was prescribed?
restored. Which enzyme deficit                     A. Dobutamine
prolongs the action of muscle relaxant?            B. Mildronate
A. Pseudocholinesterase                            C. Amyodarone
B. K-Na-adenosine triphosphatase                   D. Digoxin
C. N - acetyltransferase                           E. Dopamine
D. Succinate dehydrogenase                         98. A young man consulted a doctor about
E. Carbanhydrase                                   disturbed urination. Examination of his
93. Ammonia is a very toxic substance,             external genitals revealed that urethra is
especially for nervous system. What                split on top and urine runs out of this
substance takes the most active part 111           opening. What anomaly of external genitals
ammonia detoxication in brain tissues?             development is the case?
A. Glutamic acid                                   A. Paraphimosis
B. Alanine                                         B. Epispadia
C. Proline                                         C. Hypospadia
D. Histidine                                       D. Hermaphroditism
E. Lysine                                          E. Phimosis
94. A child's blood presents high content          99. Violation of safety rules resulted in
of galactose, glucose concentration is low.        calomel intoxication. Two days later the
There are such presentations as cataract,          daily diuresis was 620 m1. A patient
mental deficiency, adipose degeneration of         experienced headache, vomiting, convulsions,
liver. What disease is it?                         dyspnea, moist rales in lungs. What
A. Galactosemia                                    pathology is it?
B. Steroid diabetes                                A. Acute renal insufficiency
C. Lactosemia                                      B. Chronic renal insufficiency
D. Diabetes mellitus                               C. Uraemic coma
E. Fructosemia                                     D. Pyelonephritis
95. In course of an experiment a big number        E. Glomerulonephritis
of column cells of red bone marrow was in          100. A patient has a transverse disruption of
some way destructed. Regeneration of which cell    spinal cord below the IV thoracic segment.
populations in the loose connective                What changes of respiration will it cause?
A. Respiration will become more frequent                C. Melancholic
B. Respiration will stay unchanged                      D. Sanguine
C. Respiration will stop                                E. -
D. Respiration will become less frequent                106. A patient suffers from hepatic cirrhosis.
E. Respiration will become deeper                       Examination of which of the following
101. A patient with a stab wound of the                 substances excreted by urine can characterize
anterior stomach wall is in surgical care.              the state of antitoxic function of liver?
What formation of abdominal cavity did the              A. Hippuric acid
stomach contents get into?                              B. Ammonium salts
A. Right mesenteric sinus
                                                        C. Aminoacids
B. Hepatic bursa
C. Omental bursa                                        D. Kreatinine
D. Left mesenteric sinus                                E. Uric acid
E. Antegastrial bursa                                   107. Examination of a child revealed some
102. A patient with infectious mononucleosis            whitish spots looking like coagulated milk
has been taking glucocorticoids for two                 on the mucous membrane of his cheeks and
weeks. He was brought into remission, but               tongue. Analysis of smears revealed grampositive
he fell ill with acute attack of chronic tonsillitis.   oval yeast-like cells. What causative
What action of glucocorticoids caused                   agents are they?
this complication?                                      A. Actinomycetes
A. Anti-inflammatory                                    B. Staphylococci
B. Immunosuppressive                                    C. Candida
C. Antiallergic                                         D. Fusobacteria
D. Antishock                                            E. Diphtheria bacillus
E. Antitoxic                                            108. A patient's knee joint doesn't extend,
103. Removal of gall bladder of a patient               there is no knee-jerk ref1ex, skin sensitivity
has disturbed processes of Co absorption                of the anterior femoral surface is disturbed.
through the intestinal wall. What vitamin               What nerve structures are damaged?
will stimulate this process?                            A. Superior gluteal nerve
A. B12                                                  B. Femoral nerve
B. PP                                                   C. Big fibular nerve
C. C                                                    D. Inferior gluteal nerve
D. K                                                    E. Obturator nerve
E. D3                                                   109. A patient has pain, edema and reddening
104. A 9 m.o. child has delayed dentition, it           of his skin in the anterosuperior area of
is also out of order. Upper jaw configuration           his thigh and his foot's thumb. What lymph
is horizontal ("high palate); microscopically           nodes of his lower extremity responded to
- irregular mineralization of tooth enamel,             the inflammatory process?
wrinkled enamel prisms, some of them are                A. Deep inguinal
vacuolized. Predentin zone is extended;                 B. Superficial longitudinal
there are solitary denticles. What disease              C. Superficial inguinal
is it?                                                  D. lnternal longitudinal
A. Late rickets                                         E. General longitudinal
B. Early rickets                                        110. Examination of a 43 y.o. patient
C. Hypervitaminosis D                                   revealed that his stomach has difficulties with
D. Gout                                                 digestion of protein food. Gastric juice
E. Osteon1alacia                                        analysis revealed low acidity. Function of
105. Examination of a patient revealed                  which gastric cells is disturbed in this case?
a strong, balanced, inert type of higher                A. Main exocrinocytes
nervous activity according to Pavlov. What              B. Parietal exocrinocytes
temperament type does the patient have                  C. Mucous cells (mucocytes)
(according to Hiprocrates classification)?              D. Endocrinous cells
A. Phlegmatic                                           E. Cervical mucocytes
R Choleric                                              111. The permeability of the irritable cell
membrane has been increased for potassium          C. Lactin and myosin synthesis
ions during an experiment. What changes of         D. Regulation of Ca2+ rate in mitochondrions
membrane electric status can occur?                E. Substrate phosphorylation
A. Action potential                                117. A 30 y.o. woman had been ill for a
B. No changes                                      year when she felt pain in the area of joints
C. Hyperpolarization                               for the first time, they got swollen and
D. Local response                                  skin above them became reddened. Provisional
E. Depolarization                                  diagnosis is rheumatoid arthritis. One
112. An ovary specimen stained by                  of the most probable causes of this disease is
hematoxylin-eosin presents a follicle, where       a structure alteration of a connective tissue
cells of follicular epithelium are placed in       protein:
1-2 layers and have cubic form, there is a         A. Collagen
bright-red membrane around the ovocyte.            B. Troponin
What follicle is it?                               C. Ovoalbumin
A. Secondary                                       D. Mucin
B. Atretic                                         E. Myosin
C. Primordial                                      118. A 56 y.o. patient has been suffering from
D. Primary                                         thyreotoxicosis for a long time. What type of
E. Mature                                          hypoxia can be developed?
113. Utilization of arachidonic acid via           A. Tissue
cyclooxigenase pathway results in formation        B. Hemic
of some bioactive substances. Name them:           C. Circulatory
A. Prostaglandins                                  D. Respiratory
B. Biogenic amins                                  E. Mixed
C. Insulin-like growth factors                     119. A 48 y.o. patient was admitted to the
D. Somatomedins                                    hospital with complaints about weakness,
E. Thyroxine                                       irritability, sleep disturbance. Objectively:
114. A woman has been applying a new               skin and scleras are yellow. In blood:
cosmetic preparation for a week that               conjugated bilirubin, cholalemia. Feces are
resulted in eye-lid inflammation accompanied       acholic. Urine is of dark colour (bilirubin).
by hyperemia, infiltration and painfulness.        What jaundice is it?
What type of allergic reaction was                 A. Mechanic
developed?                                         B. Crigler-Najjar syndrome
A.IV                                               C. Hemolytic
B. I                                               D. Parenchymatous
C. III                                             E. Gilbert's syndrome
D.V                                                120. A lightly dressed man is standing in a
E. II                                              room, air temperature is +14°C, windows
115. A doctor administered a patient with          and doors are closed. In what way does he
allergic dermatitis a H1 -histamine blocker        emit heat the most actively?
as a part of complex treatment. Name this          A. Perspiration
medication:                                        B. Heat conduction
A. Prednisolone                                    C. Heat radiation
B. Loratadine                                      D. Convection
C. Cromolyn sodium                                 E. Evaporation
D. Hydrocortisone                                  121. A patient complains of pain in the area
E. Adrenaline                                      of his liver. Duodenal intubation revealed
116. A 1 y.o. child with symptoms of muscle        yellowish, oval, narrowed at the poles eggs
affection was admitted to the hospital.            with an operculum at the end. Size of these
Examination revealed carnitine deficit in          eggs is the smallest among all helminth eggs.
muscles. Biochemical base of this pathology        What is the most probable diagnosis?
is disturbed process of:                           A. Beef tapeworm infection
A. Transporting of fatty acids to mitochondrions   B. Diphyllobothriasis
B. Lactic acid utilization                         C. Echinococcosis
D. Teniasis                                           a worker has largely reduced buffer capacity
E. Opisthorchosis                                     of blood. What acidic substance that came
122. A patient who suffers from severe disorder       to blood caused this phenomenon?
of water-salt metabolism experienced                  A. -
cardiac arrest in diastole. What is the most          B. l,3-bisphosphoglycerate
probable mechanism of cardiac arrest in diastole?     C. 3-phosphoglycerate
A. Hypernatremia                                      D. Pyruvate
B. Hyponatremia                                       E. Lactate
C. Hypokaliemia                                       128. Bacteriological laboratory examines
D. Organism dehydratation                             canned meat whether it contains botulinum
E. Hyperkaliemia                                      toxin. For this purpose an extract of test
123. A 36 y.o. man has a craniocerebral               specimen and antitoxic antibotulinic scrum
trauma. Objectively: diminished breath                of A, B, E types were introducted to a group
sounds, thready pulse, no reflexes. What way          of mice under examination; a control group
of pyracetam introduction will be the most            of mice got the extract without antibotulinic
apropriate in this case?                              serum. What serological reaction was applied?
A. Subcutaneous                                       A. Complement binding
B. Intravenous                                        B. Precipitation
C. Inhalation                                         C. Opsono-phagocytic
D. Rectal                                             D. Neutralization
E. Peroral                                            E. Double immune diffusion
124. Short-term physical activity resulted in         129. Parents of a 10 y.o. boy consulted a
reflex amplification of heart rate and raise of       doctor about extension of hair-covering,
systemic arterial pressure. What receptors            growth of beard and moustache, low voice.
activation was the main cause of pressor              Intensified secretion of which hormone
reflex realization?                                   must be assumed?
A. Vascular chemoreceptors                            A. Of testosterone
B. Vascular volume receptors                          B. Of somatotropin
C. Proprioreceptors of active muscles                 C. Of cortisol
D. Hypothalamus thermoreceptors                       D. Of oestrogen
E. Vascular baroccptors                               E. Of progesterone
125. A patient with neuritis of femoral nerve         130. A 60 y.o. patient has a reduced
has disturbed flexion of thigh as well as disturbed   perception of high-frequency sounds. What
crus extension in the knee joint.                     structures' disorder of auditory analizer
What muscle's function is disturbed?                  caused these changes?
A. Quadriceps muscle of thigh                         A. Main membrane of cochlea near helicotrema
B. Semitendinous muscle                               B. Main membrane of cochlea near the oval
C. Triceps muscle of thigh                            window
D. Biceps muscle of thigh                             C. Muscles of middle ear
E. Semimembranous muscle                              D. Eustachian tube
126. A 55 y.o. woman consulted a doctor about         E. Tympanic membrane
having continuous cyclic uterine                      131. In course of an experiment a skeletal
hemorrhages for a year, weakness, dizziness.          muscle is being stimulated by a series of
Examination revealed skin pallor.                     electric impulses. What type of muscle
Hcmogram: Hb- 70 g/L erythrocytes 3.2*1012/l,         contraction will arise, if every subsequent
color index - 0,6, leukocytcs 6.0*109/l,              impulse comes in the period of relaxation of
reticulocytes - 1%; erythrocyte                       single muscle contraction?
hypochromia. What anemia is it?                       A. Muscle contructure
A. Hemolytic anemia                                   B. A series of single contractions
B. B12 -folate-deficiency anemia                      C. Holotetanus
C. Chronic posthemorrhagic anemia                     D. Asynchronous tetanus
D. Iron-deficiency anemia                             E. Partial tetanus
E. Aplastic anemia                                    132. A liquidator of a breakdown at a
127. As a result of exhausting muscular work          nuclear power plant who was irradiated
complained about vomiting that occurs all            E. Lisosomes
of a sudden. What medication should be               137. A 22 y.o. woman has enlarged lymph
prescribed?                                          nodes. Histologlcally: a lymph node contains
A. Metoclopramide                                    lymphocytes, histiocytes, reticular cells,
B. Aeron                                             small and big Hodgkin's cells, multinucleated
C. Reserpine                                         Sternberg cells, isolated foci of
D. De-Nol                                            caseous necrosis. What disease are these
E. Atropine                                          changes typical for?
133. A patient got a craniocerebral trauma           A. Lung cancer metastasis
that resulted in right-side convergent strabismus.   B. Chronic leukosis
Damage of which craniocerebral                       C. Acute leukosis
nerve caused such consequences?                      D. Lymphogranulomatosis
A. n.facialis                                        E. Lymphosarcoma
B. n.trigeminus                                      138. A 49 y.o. woman consulted a doctor
C. n.aculomotorius                                   about heightened fatigue and dyspnea during
D. n.trochlearis                                     physical activity. ECG: heart rate is
E. n.abducens                                        50/min, PQ is extended, QRS is unchanged,
134. A patient ill with bronchial asthma didn't      P wave quanity exceeds quantity of QRS
inform his doctor that he had attacks                complexes. What type of arrhythmia does
of stenocardia. Doctor administered him a            the patient have?
medication, which taking resulted in less            A. Extrasystole
frequent attacks of bronchial asthma. but            B. Sinoatrial block
stenocardia attacks became more frequent.            C. Sinus bradycardia
What medication was administered?                    D. Atrioventricular block
A. Cromolyn sodium                                   E. Ciliary arhythmia
B. Salbutamol                                        139. A patient who suffers from pneumonia
C. Aminophylline                                     has high body temperature. What biologically
D. Phenotherol                                       active substance plays the leading part
E. Isadrin                                           in origin of this phenomenon?
135. Autopsy of a man who died from                  A. Bradykinin
influenza revealed that his heart was slightly       B. Leukotrienes
enlarged, pastous, myocardium was                    C. Histamine
dull and had specks. Microscopical cxamination       D. Serotonin
of myocardium revealed signs of                      E. Interleukin-I
parenchymatous adipose and hydropic                  140. A teenager was irradiated with high
dystrophy; stroma was edematic with poor             radiation dose that resulted in serious
macrophagal and lymphocytic infiltration,            damages of lymphoid system, lysis of
vessels were plethoric; perivascular analysis        many lymphocytes. Restoration of normal
revealed petechial hemorrhages. What type            hemogram is possible due to the functioning
of myocarditis was developed in this case?           of the following gland:
A. Serous diffuse                                    A. Thyroid
B. Granulomatous                                     B. Adrenal
C. Purulent                                          C. Liver
D. Interstitial proliferative                        D. Thymus
E. Serous focal                                      E. Pancreas
136. In course of practical training students        141. In course of histidine catabolism a biogenic
studied a stained blood smear of a mouse with        amin is formed that has powerful
bacteria phagocyted by leukocytes. What              vasodilatating effect. Name it:
cell organella completes digestion of these          A. Histamine
bacteria?                                            B. Dopamine
A. Mytochondrions                                    C. Dioxyphenylalanine
B. Ribosomes                                         D. Noradrenalin
C. Golgi apparatus                                   E. Serotonin
D. Granular endoplasmic reticulum                    142. According to audiometry data a patient
has a disturbed perception of mediumfrequency         A. Spinothalamic
sounds. It might have been caused                     B. Medial spinocortical
by a damage of:                                       C Anterior spinocerebellar
A. Cochlear nuclei                                    D. Posterior spinocerebellar
B. Quadritubercular structure                         E. Lateral spinocortical
C. Middle part of helix                               147. A patient who suffers from heart failure
D. Lateral geniculate bodies                          has enlarged liver, edemata of lower
E. Spiral ganglion                                    extremities, ascites. What is the leading
143. A patient with fracture of his lower             mechanism in the development of this
jaw was admitted to the maxillofacial                 edema?
department. It was decided to fix his                 A. Hydrodynamic
bones surgically under anaesthetic. After             B. Membranogenic
intravenous introduction of muscle relaxant           C-
there arose short fibrillar contractions of           D. Lymphogenous
the patient's facial muscles. What muscle             E. Colloid osmotic
relaxant was applied?                                 148. A rabbit's nerve that innervates the
A. Dithylinum                                         right ear was cut and its right superior
B. Pipecuronium bromide                               cervical ganglion was removed. Immediately
C. Diazepam                                           after operation the temperature of
D. Melictine                                          ear skin was measured. It was revealed that
E. Tubocurarin chloride                               the temperature of the rabbit's ear skin on
144. Analysis of blood serum of a patient             the side of denervation was by 1,5°C higher
revealed increase of alanine                          than on the opposite intact side. What of the
aminotransferase and aspartate aminotransferase       following is the most probable explanation
level. What cytological                               of the above-mentioned effects?
changes can cause such a situation?                   A. Atrerial hyperemia induced by metabolic
A. Disturbance of cellular interrelations             factors
B. Cellular breakdown                                 B. Arterial neuroparalytic hyperemia
C Disturbance of genetic apparatus of cells           C. Arterial neurotopical hyperemia
D. Disturbed function of energy supply of             D. Reactive arterial hyperemia
cells                                                 E. Physiological arterial hyperemia
E. Disorder of enzyme systems of cells                149. Inflammation of a patient's eye was
145. A 63 y.o. man fell ill with acute tracheitis     accompanied by accumulation of turbid liquid
and bronchitis accompanied by bronchial               with high protein at the bottom of
pneumonia. On the l0th day the patient                anterior chamber that was called hypopyon.
died from cardiopulmonary insufficiency               What process underlies the changes under
Autopsy revealed fibrinous hemorrhagic                observation?
laryngotracheobronchitis: lungs were                  A. Primary alteration
enlarged. their incision revealed-the "coalminer's"   B. Proliferation
effect caused by interlacing of sections              C.-
of bronchial pneumonia. hemorrhages                   D. Secondary alteration
into the pulmonary parenchyma, acute                  E. Disturbance of microcirculation
abscesses and atelectases. Internal organs            150. A patient was ill with burn disease that
have discirculatory and dystrophic changes.           was complicated by DIC syndrome. What
What is the most probable diagnosis?                  stage of DIC syndrome can he suspected if it
A. Moderately severe influenza                        IS known that the patient's blood coagulates
B. Adenoviral infection                               in less than 3 minutes?
C Parainfluenza                                       A. Terminal
D. Influenza. severe form                             B. Transition phase
E. Respiratory syncytial infection                    C. Hypocoagulation
146. As a result of spinal-cord trauma                D. Hypercoagulation
a 33 y.o. man has a disturbed pain and                E. Fibrinolysis
temperature sensitivity that is caused by             151. An experimental animal has been given
damage of the following tract:                        excessive amount of carbon-Iabeled glucose
for a week. What compound can the label           B. Supracutaneous tularin test
be found in?                                      C. Mantoux test
A. Methionine                                     D. Burnet test
B. Arachidonic acid                               E. Anthraxine test
C. Palmitic acid                                  157. A patient has extrasystole. ECG shows
D. Choline                                        no P wave, QRS complex is deformed,
E. Vitamin A                                      there is a full compensatory pause. What
152. A patient complains of dryness of            extrasystoles are these?
head skin, itching, fragility and loss of hair.   A. Atrioventricular
After examination he was diagnosed with           B. Sinus
seborrhea. Disturbed activity of which cells      C. Ventricular
caused this condition?                            D. Atrial
A. Cells of sudoriferous glands                   E. -
B. Melanocytes                                    158. After intake of rich food a patient
C. Adipocytes                                     feels nausea and sluggishness; with time
D. Epithelial cells                               there appeared signs of steatorrhea.
E. Cells of sebaceous glands                      Blood cholesterine concentration is 9,2
153. A patient who had been working hard          micromole/l. This condition was caused by lack
under conditions of elevated temperature          of:
of the environment, has now a changed             A. Fatty acids
quantity of blood plasma proteins. What           B. Bile acids
penomenon is the case?                            C. Phospholipids
A. Paraproteinemia                                D. Chylomicrons
B. Absolute hyperproteinemia                      E. Triglycerides
C. Disproteinemia                                 159. A 27 y.o. patient put eye drops
D. Relative hyperproteinemia                      that contain penicillin. After a few minutes
E. Absolute hypoproteinemia                       she felt itching and burning of
154. Examination of a patient revealed            her body, there appeared lip and eye-lid
extremely myotic pupils, sleepiness,              edemata; arterial pressure began to drop.
infrequent Chain-Stoke's respiration. urinary     What immunoglobulins took part in the
retention, slowing-down of heart                  development of this allergic reaction?
rate, enhancement of spinal reflexes. What        A. IgE and IgG
substance caused the poisoning?                   B. IgG and IgD
A. Atropine                                       C. IgM and IgD
B. Phosphacole                                    D. IgM and IgG
C. Caffeine                                       E. IgA and IgM
D. Morphine                                       160. A patient suffers from severe
E. Barbital                                       postoperative pseudomonadous infection.
155. Examination of a patient with frequent       What of the following antibiotics should be
hemorrhages from internals and mucous             administered in this case?
membranes revealed proline and lysine             A. Erythromycin
being a part of collagene fibers. What vitamin    B. Doxycycline
absence caused disturbance of their               C. Cephazolin
hydroxylation?                                    D. Benzylpenicillin
A. Vitamin K                                      E. Amicacin sulfate
B. Vitamin E                                      161. A patient complains of frequent
C. Vitamin A                                      diarrheas, especially after consumption
D. Vitamin e                                      of fattening food, and of body weight
E. Thiamine                                       loss. Laboratory examination revealed
156. The first grade pupils were examined in      steatorrhea: hypocholic feces. What can be
order to sort out children for tuberculosis       the cause of this condition?
revaccination. What test was applied for this     A. Mucous membrane inflammation of small
purpose?                                          intestine
A. Schick test                                    B. Unbalanced diet
C. Lack of pancreatic phospholipase                 C. Manitole
D. Obturation of biliary tracts                     D. Clopamide
E. Lack of pancreatic lipase                        E. Amyloride
162. Having helped to eliminate                     167. A newborn child with pylorostenosis
consequences of a failure at a nuclear              has often repeating vomiting accompanied
power plant, a worker got an irradiation            by apathy, weakness, hypertonicity, sometimes
doze of 500 roentgen. He complains of               convulsions. What disorder form of
headache, nausea, dizziness. What changes           acid-base balance is it?
in leukocytes quantity can be expected 10           A. Excretory acidosis
hours after irradiation?                            B. Nongaseous alkalosis
A. Neutrophilic leukocytosis                        C. Gaseous acidosis
B. Agranulocytosis                                  D. Metabolic acidosis
C. Leukopenia                                       E. Gaseous alkalosis
D. Leukemia                                         168. After a 2 y.o. child has had flu, there
E. Lymphocytosis                                    appeared complaints about ear ache. A
163. In case of a penetrating wound of the          doctor revealed hearing impairment and
anterior abdominal wall the wound tract             inflammation of the middle ear. How did
went above the lesser curvature of stomach.         the infection penetrate into the middle ear?
What peritoneum formation is most likely            A. Through the auditory tube
to be injured?                                      B. Through foramen jugu1aris
A. Ligamentum triangulare sinistrum                 C. Through canalis caroticus
B. Ligamentum hepatorenale                          D. Through canalis nasolacrimalis
C. Ligamentum gastrocolicum                         E. Through atrium mastoideum
D. Ligamentum hepatogastricum                       169. In course of an experiment a skeletal
E. Ligamentum hepatoduoduodenale                    muscle is being stimulated by a series of
164. An electron micrograph of a kidney             electric impulses. What type of muscle
fragment presents an afferent arteriole.            contraction will arise, if every subsequent
Under its endothelium some big cells can be         impulse comes in the period of shortening
seen that contain secretory granules. What          of the previous single muscle contraction?
type of cells is it?                                A. Holotetanus
A. Juxtaglomerular                                  B. Partial tetanus
B. Smooth muscle cells                              C. A series of single contractions
C. Interstitial                                     D. Muscle contracture
D. Juxtavascular                                    E. Asynchronous tetanus
E. Mesangeal                                        170. Examination of a miner revealed
165. Arterial pressure of a surgeon who             pulmonary fibrosis accompanied by disturbance
performed a long operation rised up to              of alveolar ventilation. What is
140/110 mm Hg. What changes of humoral              the main mechanism of this disturbance?
regulation could have caused the rise of            A. Limitation of respiratory surface of lungs
arterial pressure in this case?                     B. Constriction of superior respiratory tracts
A. Activation of renin angiotensive system          C. Bronchi spasm
B. Activation of sympathoadrenal system             D. Disturbance of neural respiration control
C. Inhibition of sympathoadrenal system             E. Limitation of breast mobility
D. Activation of formation and excretion of         171. A patient has symptoms of inflammation
aldosterone                                         of urogenital tracts. Examination of a
E. Activation of kallikrein kinin system            vaginal smear revealed big monocellular,
166. A patient with chronic cardiac insufficiency   pear-shaped organisms with the pointed spike
has been treated with cardiotonic drugs             at the posterior end of body, big nucleus
and a thiazide diuretic, but in spite of it there   and undulating membrane. What protozoa
are still edemata and risk of ascites. What         were found in the smear?
medication should be prescribed to amplify          A. Trypanosoma gambiense
diuretic effect of the applied drugs?               B. Trichomonas vaginalis
A. Spironolactone                                   C. Trichomonas hominis
B. Furosemide                                       D. Trichomonas buccalis
E. Lamblia interstinalis                          B. Verapamil
172. A child is languid, apathetic. Liver         C. Nitroglycerin
is enlarged and liver biopsy revealed a           D. Metoprolol
significant excess of glycogene. Glucose          E. Propranolol
concentration in the blood stream is below        177. A 63 y.o. man with collapse symptoms
normal. What is the cause of low glucose          was delivered to the emergency hospital.
concentration?                                    A doctor chose noradrenaline in order to
A. Deficit of a gene that is responsible          prevent hypotension. What is the action
for synthesis of glucose 1-phosphaturidine        mechanism of this medication?
transferase                                       A. Activation of dopamine receptors
B. Low (absent) activity of hexokinase            B. Block of M -cholinoreceptors
C. High activity of glycogen synthetase           C. Activation of serotonin receptors
D. Low (absent) activity of glucose 6-            D. Activation of α1-adrenoreceptors
phosphatase                                       E. Activation of β-adrenoreceptors
E. Low (absent) activity of glycogene             178. Urine examination of a patient with
phosphorylase in liver                            acute cystitis revealed leukocytes and
173. A man who went for a ride on a               a lot of gram-negative bacilli. Inoculation
roundabout had amplification of heart rate,       resulted in growth of colonies of mucous
sweating and nausea. What receptors stimulation   nature that formed green soluble pigment.
is it primarily connected with?                   What microorganism is the most probable
A. Proprioceptors                                 cause of the disease?
B. Visual                                         A. Proteus mirabilis
C. Auditory                                       B. Escherihia coli
D. Tactors                                        C. Salmonella enteritidis
E. Vestibular                                     D. Klebsiella pneumoniae
174. Two days after consumption of                E. Pseudomonas aeruginosa
smoked pork a patient got face and eyelid         179. An isolated cell of human heart
edemata, gastrointestinal disturbances,           automatically generates excitation impulses
abrupt temperature rise, muscle pain. Blood       with frequency 60 times pro minute. What
analysis showed full-blown eosinophilia.          heart structure was this cell obtained from?
What helminth could the patient be infected       A. Ventricle
with?                                             B. Atrium
A. Trichina                                       C. Sinoatrial node
B. Hookworm                                       D. Atrioventricular node
C. Whipworm                                       E. His' bundle
D. Ascarid                                        180. A patient died under conditions of
E. Pinworm                                        cardiovascular insufficiency. Autopsy results:
175. Examination of a patient revealed            postinfarction cardiosclcrosis, myocardium
hyperkaliemia and hyponatremia. Low               hypertrophy and dilatation of its cavities,
secretion of which hormone may cause such         especially of its right ventricle. Liver
changes?                                          is enlarged, its surface is smooth, incision
A. Natriuretic                                    revealed that it was plethoric, with dark-red
B. Parathormone                                   specks against the background of brownish
C. Cortisol                                       tissue. Histologically: plethora of central
D. Aldosteron                                     parts of lobules; peritheral parts around
E. Vasopressin                                    portal tracts contain hepatocytes in a state
176. After a tooth extraction a patient felt      of adipose degeneration. How are these liver
persistent pain behind his breast bone. After     changes called?
sublingual intake of an antianginal drug the      A. Nutmeg liver
pain behind the breast bone disappeared,          B. Liver cirrhosis
but the patient complained of headache and        C. Liver steatosis
dizziness. What drug are these properties         D. Pseudonutmeg liver
typical for?                                      E. Amyloidosis
A. Validol                                        181. Autopsy of a newborn boy revealed
polydactylia, microcephalia, cheiloschisis          D. Acute erythromyelosis
and uranoschisis as well as hypertrophy             E. Acute myeloblastic leukosis
of parenchimatous organs. These defects             186. Continious taking of a drug can result
correspond with the description of Patau's          in osteoporosis. erosion of stomach mucous
syndrome. What is the most probable cause           membrane, hypokaliemia, retention of sodium
of this pathology?                                  and water, reduced content of corticotropin
A. Trisomy of the 18th chromosome                   in blood. Name this drug:
B. Trisomy of the 21st chromosome                   A. Digoxin
C. Trisomy of the 13th chromosome                   B. Indometacin
D. Nondisjunction of sex chromosomes                C. Reserpine
E. Partial monosomy                                 D. Prednisolone
182. Glutamate decarboxylation results in           E. Hydrochlorothiazide
formation of inhibitory transmitter in CNS.         187. A patient has a haemorrhage into the
Name it:                                            posterior central gyrus. What type of sensitivity
A. Serotonin                                        on the opposite side will be disturbed?
B. Histamine                                        A. Visual
C. Glutathione                                      B. Auditory and visual
D. Asparagine                                       C. Auditory
E. GABA                                             D. Olfactory
183. A patient who suffers from acute               E. Skin and proprioceptive
myocarditis has clinical signs of cardiogenic       188. As a result of an accident a patient
shock. What of the under-mentioned                  has intense painfullness and edema of the
pathogenetic mechanisms plays the main              anterior crus surface: dorsal f1exion of foot
part in shock development?                          is hindered. Function of which crus muscle
A. Disturbance of pumping ability of heart          is most likely to be disturbed?
B. Depositing of blood in organs                    A. M.peroneus longus
C. Reduction of diastolic flow to the heart         B. M.flexor digitorum longus
D. Decrease of vascular tone                        C. M.flexor hallucis longus
E. Increase of peripheral vascular resistance       D. M.peroneus brevis
184. A patient who has been treated                 E. M.tibialis anterior
with diazepam on account of neurosis                189. A 2 y.o. child has convulsions as a result
complains of toothache. Doctor administered         of lowered concentration of calcium ions in
him an analgetic, but its dose was lower than       blood plasma. It is caused by reduced function
average therapeutic dose. What                      of:
phenomenon did the doctor take into                 A. Adrenal cortex
account while prescribing the patient an            B. Pineal gland
underdose?                                          C. Hypophysis
A. Summation                                        D. Parathyroid glands
B. Potentiation                                     E. Thymus
C. Cumulation                                       190. Histological specimen presents a vessel
D. Tolerance                                        the wall of which consists of endothelium,
E. Drug dependence                                  basal membrane and loose connective tissue.
185. A 23 y.o. patient complains of                 What type of vessel is it?
weakness, temperature rise up to 38-40°C.           A. Hemocapillary
Objectively: liver and spleen are enlarged.         B. Lymphocapillary
Hemogram: Hb- 100 g/L erythrocytes -                C. Vein of non-muscular type
2.9*1012/l, leukocvtes - 4.4· l09/l, thrombocytes   D. Vein of muscular type
- 48*109 /1, segmentonuclear neutrophils 17%        E. Artery
lymphocytes - 15%, blast cells - 68%.               191. Examination of a man who hadn't been
All cytochemical reactions are negative.            consuming fats but had been getting enough
Make a hematological conclusion:                    carbohydrates and proteins for a long time
A. Undifferentiated leukosis                        revealed dermatitis, poor wound healing, vision
B. Acute lymphoblastic leukosis                     impairment. What is the probable cause
C. Chronic myeloleukosis                            of metabolic disorder?
A. Low caloric value of diet                         necrosis, fibrinous thrombs in their lumens;
B. Lack of vitamins PP, H                            sclerosis and hyalinosis of glomerules,
C. Lack of palmitic acid                             atrophy of tubules and fibrosis of renal
D. Lack of linoleic acid, vitamins A, D, E, K        stroma. What is the most probable diagnosis?
E. Lack of oleic acid                                A. Membranous nephropathy
192. Analysis of a punction biopsy material          B. Acute glomerulonephritis
of liver revealed hepatocyte dystrophy with          C. Focal segmentary sclerosis
necroses as well as sclerosis with disorder of       D. Subacute glomerulonephritis
beam and lobulous structure, with formation          E. Chronic glomerulonephritis
of pseudolobules and regenerative nodes.             197. To prevent postoperative bleeding
What is the most probable diagnosis:                 a 6 y.o. child was administered vicasol
A. Acute hepatitis                                   that is a synthetic analogue of vitamin K.
B. Progressive massive liver necrosis                Name post-translational changes of blood
C. Liver cirrhosis                                   coagulation factors that will be activated by
D. Chronic hepatosis                                 vicasol:
E. Chronic hepatitis                                 A. Phosphorylation of serine radicals
193. A 50 y.o. patient was admitted to the           B. Polymerization
hospital with complaints about pain behind           C. Partial proteolysis
his breastbone, asphyxia during physical             D. Glycosyiation
activity. Angiography revealed pathological          E. Carboxylation of glutamin acid
changes in the posterior interventricular            198. According to clinical indications a patient
branch of the right eoronary artery. What            was administered pyridoxal phosphate.
heart parts are affected?                            What processes is this medication intended
A. Posterior wall of the right and left ventricles   to correct?
B. Right atrioventricular valve                      A. Oxidative decarboxylation of ketonic
C. Right atrium                                      acids
D. Left atrium                                       B. Transamination and decarboxylation of
E. Anterior wall of the right and left ventricles    aminoacids
194. A hepatitis outbreak was registered in          C. Desamination of purine nucleotide
a settlement. This episode is connected with         D. Synthesis of purine and pyrimidine bases
water factor. What hepatitis virus could             E. Protein synthesis
have caused the infective outbreak in this           199. A 62 y.o. woman complains of
settlement?                                          frequent pains in the area of her chest and
A.E                                                  backbone, rib fractures. A doctor assumed
B.C                                                  myelomatosis (plasmocytoma). What of the
C.B                                                  following laboratory characteristics will be
D.D                                                  of the greatest diagnostical importance?
E.G                                                  A. Hyperalbuminemia
195. Examination of coronary arteries                B. Proteinuria
revealed atherosclerotic calcific plaques that       C. Hypoglobulinemia
close vessel lumen by 1/3. The muscle has            D. Paraproteinemia
multiple whitish layers of connective tissue.        E. Hypoproteinemia
What process was revealed in myocardium?             200. A hypertensive glucose solution was
A. Postinfarction cardiosclerosis                    introduced to a patient. It will intensify
B. Tiger heart                                       water movement:
C. Myocardium infarction                             A. From the capillaries to the intercellular
D. Myocarditis                                       liquid
E. Diffuse cardiosclerosis                           B. There will be no changes of water
196. Microscopical renal examination of a            movement
36 y.o. woman who died from renal insufficiency      C. From the intercellular liquid to the cells
revealed in the glomerules proliferation             D. From the cells to the intercellular liquid
of capsule nephrothelium as well as                  E. From the intercellular liquid to the capillaries.
of podocytes and phagocytes accompanied
by formation of "crescents", capillary loop

More Related Content

What's hot

Krok 1 - 2010 Question Paper (Pharmacy)
Krok 1 - 2010 Question Paper (Pharmacy)Krok 1 - 2010 Question Paper (Pharmacy)
Krok 1 - 2010 Question Paper (Pharmacy)Eneutron
 
Krok 1 - 2014 Question Paper (Pharmacy)
Krok 1 - 2014 Question Paper (Pharmacy)Krok 1 - 2014 Question Paper (Pharmacy)
Krok 1 - 2014 Question Paper (Pharmacy)Eneutron
 
Krok 1 - 2011 Question Paper (Pharmacy)
Krok 1 - 2011 Question Paper (Pharmacy)Krok 1 - 2011 Question Paper (Pharmacy)
Krok 1 - 2011 Question Paper (Pharmacy)Eneutron
 
Krok 1 - 2017 Question Paper (Pharmacy)
Krok 1 - 2017 Question Paper (Pharmacy)Krok 1 - 2017 Question Paper (Pharmacy)
Krok 1 - 2017 Question Paper (Pharmacy)Eneutron
 
Krok 1 - 2015 Question Paper (Pharmacy)
Krok 1 - 2015 Question Paper (Pharmacy)Krok 1 - 2015 Question Paper (Pharmacy)
Krok 1 - 2015 Question Paper (Pharmacy)Eneutron
 
Krok 1 - 2008 Question Paper (Pharmacy)
Krok 1 - 2008 Question Paper (Pharmacy)Krok 1 - 2008 Question Paper (Pharmacy)
Krok 1 - 2008 Question Paper (Pharmacy)Eneutron
 
Krok 1 - 2007 Question Paper (Pharmacy)
Krok 1 - 2007 Question Paper (Pharmacy)Krok 1 - 2007 Question Paper (Pharmacy)
Krok 1 - 2007 Question Paper (Pharmacy)Eneutron
 
Krok 1 - 2016 Question Paper (Pharmacy)
Krok 1 - 2016 Question Paper (Pharmacy)Krok 1 - 2016 Question Paper (Pharmacy)
Krok 1 - 2016 Question Paper (Pharmacy)Eneutron
 
Krok 1 - 2012 Question Paper (Pharmacy)
Krok 1 - 2012 Question Paper (Pharmacy)Krok 1 - 2012 Question Paper (Pharmacy)
Krok 1 - 2012 Question Paper (Pharmacy)Eneutron
 
Previous year question on glycolysis based on neet pg, usmle, plab and fmge o...
Previous year question on glycolysis based on neet pg, usmle, plab and fmge o...Previous year question on glycolysis based on neet pg, usmle, plab and fmge o...
Previous year question on glycolysis based on neet pg, usmle, plab and fmge o...Medico Apps
 
Obesity- Metabolic alterations, complications and treatment
Obesity- Metabolic alterations, complications and treatmentObesity- Metabolic alterations, complications and treatment
Obesity- Metabolic alterations, complications and treatmentNamrata Chhabra
 
Metabolic response to injury
Metabolic response to injuryMetabolic response to injury
Metabolic response to injurySudarsan Agarwal
 
2.mnt for metabolic stress burn...
2.mnt for metabolic stress burn...2.mnt for metabolic stress burn...
2.mnt for metabolic stress burn...Wan Hazirah
 
Metabolic respons to injury
Metabolic respons to injuryMetabolic respons to injury
Metabolic respons to injuryNabarun Biswas
 
Unexplained severe-lactacidosis-in-emergency-medicine
Unexplained severe-lactacidosis-in-emergency-medicineUnexplained severe-lactacidosis-in-emergency-medicine
Unexplained severe-lactacidosis-in-emergency-medicineAnnex Publishers
 
Physiological Responses to Surgery & Trauma
Physiological Responses to Surgery & TraumaPhysiological Responses to Surgery & Trauma
Physiological Responses to Surgery & TraumaMuhammad Shoyab
 

What's hot (20)

Krok 1 - 2010 Question Paper (Pharmacy)
Krok 1 - 2010 Question Paper (Pharmacy)Krok 1 - 2010 Question Paper (Pharmacy)
Krok 1 - 2010 Question Paper (Pharmacy)
 
Biochemistry spotters
Biochemistry spottersBiochemistry spotters
Biochemistry spotters
 
Krok 1 - 2014 Question Paper (Pharmacy)
Krok 1 - 2014 Question Paper (Pharmacy)Krok 1 - 2014 Question Paper (Pharmacy)
Krok 1 - 2014 Question Paper (Pharmacy)
 
Krok 1 - 2011 Question Paper (Pharmacy)
Krok 1 - 2011 Question Paper (Pharmacy)Krok 1 - 2011 Question Paper (Pharmacy)
Krok 1 - 2011 Question Paper (Pharmacy)
 
Krok 1 - 2017 Question Paper (Pharmacy)
Krok 1 - 2017 Question Paper (Pharmacy)Krok 1 - 2017 Question Paper (Pharmacy)
Krok 1 - 2017 Question Paper (Pharmacy)
 
Krok 1 - 2015 Question Paper (Pharmacy)
Krok 1 - 2015 Question Paper (Pharmacy)Krok 1 - 2015 Question Paper (Pharmacy)
Krok 1 - 2015 Question Paper (Pharmacy)
 
2009
20092009
2009
 
Krok 1 - 2008 Question Paper (Pharmacy)
Krok 1 - 2008 Question Paper (Pharmacy)Krok 1 - 2008 Question Paper (Pharmacy)
Krok 1 - 2008 Question Paper (Pharmacy)
 
Krok 1 - 2007 Question Paper (Pharmacy)
Krok 1 - 2007 Question Paper (Pharmacy)Krok 1 - 2007 Question Paper (Pharmacy)
Krok 1 - 2007 Question Paper (Pharmacy)
 
Krok 1 - 2016 Question Paper (Pharmacy)
Krok 1 - 2016 Question Paper (Pharmacy)Krok 1 - 2016 Question Paper (Pharmacy)
Krok 1 - 2016 Question Paper (Pharmacy)
 
Krok 1 - 2012 Question Paper (Pharmacy)
Krok 1 - 2012 Question Paper (Pharmacy)Krok 1 - 2012 Question Paper (Pharmacy)
Krok 1 - 2012 Question Paper (Pharmacy)
 
Previous year question on glycolysis based on neet pg, usmle, plab and fmge o...
Previous year question on glycolysis based on neet pg, usmle, plab and fmge o...Previous year question on glycolysis based on neet pg, usmle, plab and fmge o...
Previous year question on glycolysis based on neet pg, usmle, plab and fmge o...
 
ACES: CV Drugs / Pesticides
ACES: CV Drugs / PesticidesACES: CV Drugs / Pesticides
ACES: CV Drugs / Pesticides
 
ACES: Constipation / Thyroid-Adrenal / Elderly
ACES: Constipation / Thyroid-Adrenal / ElderlyACES: Constipation / Thyroid-Adrenal / Elderly
ACES: Constipation / Thyroid-Adrenal / Elderly
 
Obesity- Metabolic alterations, complications and treatment
Obesity- Metabolic alterations, complications and treatmentObesity- Metabolic alterations, complications and treatment
Obesity- Metabolic alterations, complications and treatment
 
Metabolic response to injury
Metabolic response to injuryMetabolic response to injury
Metabolic response to injury
 
2.mnt for metabolic stress burn...
2.mnt for metabolic stress burn...2.mnt for metabolic stress burn...
2.mnt for metabolic stress burn...
 
Metabolic respons to injury
Metabolic respons to injuryMetabolic respons to injury
Metabolic respons to injury
 
Unexplained severe-lactacidosis-in-emergency-medicine
Unexplained severe-lactacidosis-in-emergency-medicineUnexplained severe-lactacidosis-in-emergency-medicine
Unexplained severe-lactacidosis-in-emergency-medicine
 
Physiological Responses to Surgery & Trauma
Physiological Responses to Surgery & TraumaPhysiological Responses to Surgery & Trauma
Physiological Responses to Surgery & Trauma
 

Viewers also liked (19)

2005
20052005
2005
 
2010
20102010
2010
 
Diary
Diary Diary
Diary
 
Acute pancreatitis
Acute pancreatitisAcute pancreatitis
Acute pancreatitis
 
Urology
UrologyUrology
Urology
 
Larissa julielle
Larissa julielleLarissa julielle
Larissa julielle
 
Presentacion derechos reales uasd
Presentacion derechos reales uasdPresentacion derechos reales uasd
Presentacion derechos reales uasd
 
Golf
GolfGolf
Golf
 
BNZ Weekly Overview March 10 2011
BNZ Weekly Overview March 10 2011BNZ Weekly Overview March 10 2011
BNZ Weekly Overview March 10 2011
 
Presentation1
Presentation1Presentation1
Presentation1
 
Worksheet warm up games 1º ESO
Worksheet warm up games 1º ESOWorksheet warm up games 1º ESO
Worksheet warm up games 1º ESO
 
2008
20082008
2008
 
Sport medicine
Sport medicineSport medicine
Sport medicine
 
2006
20062006
2006
 
架构风格与基于网络的软件架构设计
架构风格与基于网络的软件架构设计架构风格与基于网络的软件架构设计
架构风格与基于网络的软件架构设计
 
Rg pp presentation_template
Rg pp presentation_templateRg pp presentation_template
Rg pp presentation_template
 
Ayuda1
Ayuda1Ayuda1
Ayuda1
 
marco antonio gutierrez moreno
marco antonio gutierrez morenomarco antonio gutierrez moreno
marco antonio gutierrez moreno
 
Debit terus,overdraft,pajakan group 1
Debit terus,overdraft,pajakan group 1Debit terus,overdraft,pajakan group 1
Debit terus,overdraft,pajakan group 1
 

Similar to 2007

Krok 1 - 2007 Question Paper (General medicine)
Krok 1 - 2007 Question Paper (General medicine)Krok 1 - 2007 Question Paper (General medicine)
Krok 1 - 2007 Question Paper (General medicine)Eneutron
 
Krok 1 - 2015 (Histology)
Krok 1 - 2015 (Histology)Krok 1 - 2015 (Histology)
Krok 1 - 2015 (Histology)Eneutron
 
Krok 1 - 2015 Question Paper (General medicine)
Krok 1 - 2015 Question Paper (General medicine)Krok 1 - 2015 Question Paper (General medicine)
Krok 1 - 2015 Question Paper (General medicine)Eneutron
 
Krok 1 Medicine - 2016 General Medicine
Krok 1 Medicine - 2016 General MedicineKrok 1 Medicine - 2016 General Medicine
Krok 1 Medicine - 2016 General MedicineEneutron
 
Krok 1 - 2011 Question Paper (General medicine)
Krok 1 - 2011 Question Paper (General medicine)Krok 1 - 2011 Question Paper (General medicine)
Krok 1 - 2011 Question Paper (General medicine)Eneutron
 
Krok 1 - 2014 Question Paper (General medicine)
Krok 1 - 2014 Question Paper (General medicine)Krok 1 - 2014 Question Paper (General medicine)
Krok 1 - 2014 Question Paper (General medicine)Eneutron
 
Krok 1 - 2009 Question Paper (General medicine)
Krok 1 - 2009 Question Paper (General medicine)Krok 1 - 2009 Question Paper (General medicine)
Krok 1 - 2009 Question Paper (General medicine)Eneutron
 
Krok 1 - 2014 (Path-Physiology)
Krok 1 - 2014 (Path-Physiology)Krok 1 - 2014 (Path-Physiology)
Krok 1 - 2014 (Path-Physiology)Eneutron
 
Krok 1 - 2012 Question Paper (General medicine)
Krok 1 - 2012 Question Paper (General medicine)Krok 1 - 2012 Question Paper (General medicine)
Krok 1 - 2012 Question Paper (General medicine)Eneutron
 
Krok 1 - 2013 Question Paper (General medicine)
Krok 1 - 2013 Question Paper (General medicine)Krok 1 - 2013 Question Paper (General medicine)
Krok 1 - 2013 Question Paper (General medicine)Eneutron
 
Krok 1 - 2015 (Path-Physiology)
Krok 1 - 2015 (Path-Physiology)Krok 1 - 2015 (Path-Physiology)
Krok 1 - 2015 (Path-Physiology)Eneutron
 
Krok 1 - 2015 (Biology)
Krok 1 - 2015 (Biology)Krok 1 - 2015 (Biology)
Krok 1 - 2015 (Biology)Eneutron
 
Krok 1 - 2006 Question Paper (General medicine)
Krok 1 - 2006 Question Paper (General medicine)Krok 1 - 2006 Question Paper (General medicine)
Krok 1 - 2006 Question Paper (General medicine)Eneutron
 
Krok 1 - 2005 Question Paper (General medicine)
Krok 1 - 2005 Question Paper (General medicine)Krok 1 - 2005 Question Paper (General medicine)
Krok 1 - 2005 Question Paper (General medicine)Eneutron
 
Krok1 stomatology - 2016
Krok1   stomatology - 2016Krok1   stomatology - 2016
Krok1 stomatology - 2016Eneutron
 
Krok 2 - 2013 (Pediatrics)
Krok 2 - 2013 (Pediatrics)Krok 2 - 2013 (Pediatrics)
Krok 2 - 2013 (Pediatrics)Eneutron
 
Krok 1 - 2015 Base (General Medicine)
Krok 1 - 2015 Base (General Medicine)Krok 1 - 2015 Base (General Medicine)
Krok 1 - 2015 Base (General Medicine)E_neutron
 
Krok 1 - 2015 Base (General Medicine)
Krok 1 - 2015 Base (General Medicine)Krok 1 - 2015 Base (General Medicine)
Krok 1 - 2015 Base (General Medicine)Eneutron
 
Krok 1 - 2015 (Physiology)
Krok 1 - 2015 (Physiology)Krok 1 - 2015 (Physiology)
Krok 1 - 2015 (Physiology)Eneutron
 

Similar to 2007 (20)

Krok 1 - 2007 Question Paper (General medicine)
Krok 1 - 2007 Question Paper (General medicine)Krok 1 - 2007 Question Paper (General medicine)
Krok 1 - 2007 Question Paper (General medicine)
 
Krok 1 - 2015 (Histology)
Krok 1 - 2015 (Histology)Krok 1 - 2015 (Histology)
Krok 1 - 2015 (Histology)
 
Krok 1 - 2015 Question Paper (General medicine)
Krok 1 - 2015 Question Paper (General medicine)Krok 1 - 2015 Question Paper (General medicine)
Krok 1 - 2015 Question Paper (General medicine)
 
Krok 1 Medicine - 2016 General Medicine
Krok 1 Medicine - 2016 General MedicineKrok 1 Medicine - 2016 General Medicine
Krok 1 Medicine - 2016 General Medicine
 
Krok 1 - 2011 Question Paper (General medicine)
Krok 1 - 2011 Question Paper (General medicine)Krok 1 - 2011 Question Paper (General medicine)
Krok 1 - 2011 Question Paper (General medicine)
 
Krok 1 - 2014 Question Paper (General medicine)
Krok 1 - 2014 Question Paper (General medicine)Krok 1 - 2014 Question Paper (General medicine)
Krok 1 - 2014 Question Paper (General medicine)
 
Krok 1 - 2009 Question Paper (General medicine)
Krok 1 - 2009 Question Paper (General medicine)Krok 1 - 2009 Question Paper (General medicine)
Krok 1 - 2009 Question Paper (General medicine)
 
file
file file
file
 
Krok 1 - 2014 (Path-Physiology)
Krok 1 - 2014 (Path-Physiology)Krok 1 - 2014 (Path-Physiology)
Krok 1 - 2014 (Path-Physiology)
 
Krok 1 - 2012 Question Paper (General medicine)
Krok 1 - 2012 Question Paper (General medicine)Krok 1 - 2012 Question Paper (General medicine)
Krok 1 - 2012 Question Paper (General medicine)
 
Krok 1 - 2013 Question Paper (General medicine)
Krok 1 - 2013 Question Paper (General medicine)Krok 1 - 2013 Question Paper (General medicine)
Krok 1 - 2013 Question Paper (General medicine)
 
Krok 1 - 2015 (Path-Physiology)
Krok 1 - 2015 (Path-Physiology)Krok 1 - 2015 (Path-Physiology)
Krok 1 - 2015 (Path-Physiology)
 
Krok 1 - 2015 (Biology)
Krok 1 - 2015 (Biology)Krok 1 - 2015 (Biology)
Krok 1 - 2015 (Biology)
 
Krok 1 - 2006 Question Paper (General medicine)
Krok 1 - 2006 Question Paper (General medicine)Krok 1 - 2006 Question Paper (General medicine)
Krok 1 - 2006 Question Paper (General medicine)
 
Krok 1 - 2005 Question Paper (General medicine)
Krok 1 - 2005 Question Paper (General medicine)Krok 1 - 2005 Question Paper (General medicine)
Krok 1 - 2005 Question Paper (General medicine)
 
Krok1 stomatology - 2016
Krok1   stomatology - 2016Krok1   stomatology - 2016
Krok1 stomatology - 2016
 
Krok 2 - 2013 (Pediatrics)
Krok 2 - 2013 (Pediatrics)Krok 2 - 2013 (Pediatrics)
Krok 2 - 2013 (Pediatrics)
 
Krok 1 - 2015 Base (General Medicine)
Krok 1 - 2015 Base (General Medicine)Krok 1 - 2015 Base (General Medicine)
Krok 1 - 2015 Base (General Medicine)
 
Krok 1 - 2015 Base (General Medicine)
Krok 1 - 2015 Base (General Medicine)Krok 1 - 2015 Base (General Medicine)
Krok 1 - 2015 Base (General Medicine)
 
Krok 1 - 2015 (Physiology)
Krok 1 - 2015 (Physiology)Krok 1 - 2015 (Physiology)
Krok 1 - 2015 (Physiology)
 

Recently uploaded

Measures of Position DECILES for ungrouped data
Measures of Position DECILES for ungrouped dataMeasures of Position DECILES for ungrouped data
Measures of Position DECILES for ungrouped dataBabyAnnMotar
 
4.18.24 Movement Legacies, Reflection, and Review.pptx
4.18.24 Movement Legacies, Reflection, and Review.pptx4.18.24 Movement Legacies, Reflection, and Review.pptx
4.18.24 Movement Legacies, Reflection, and Review.pptxmary850239
 
USPS® Forced Meter Migration - How to Know if Your Postage Meter Will Soon be...
USPS® Forced Meter Migration - How to Know if Your Postage Meter Will Soon be...USPS® Forced Meter Migration - How to Know if Your Postage Meter Will Soon be...
USPS® Forced Meter Migration - How to Know if Your Postage Meter Will Soon be...Postal Advocate Inc.
 
HỌC TỐT TIẾNG ANH 11 THEO CHƯƠNG TRÌNH GLOBAL SUCCESS ĐÁP ÁN CHI TIẾT - CẢ NĂ...
HỌC TỐT TIẾNG ANH 11 THEO CHƯƠNG TRÌNH GLOBAL SUCCESS ĐÁP ÁN CHI TIẾT - CẢ NĂ...HỌC TỐT TIẾNG ANH 11 THEO CHƯƠNG TRÌNH GLOBAL SUCCESS ĐÁP ÁN CHI TIẾT - CẢ NĂ...
HỌC TỐT TIẾNG ANH 11 THEO CHƯƠNG TRÌNH GLOBAL SUCCESS ĐÁP ÁN CHI TIẾT - CẢ NĂ...Nguyen Thanh Tu Collection
 
Keynote by Prof. Wurzer at Nordex about IP-design
Keynote by Prof. Wurzer at Nordex about IP-designKeynote by Prof. Wurzer at Nordex about IP-design
Keynote by Prof. Wurzer at Nordex about IP-designMIPLM
 
Visit to a blind student's school🧑‍🦯🧑‍🦯(community medicine)
Visit to a blind student's school🧑‍🦯🧑‍🦯(community medicine)Visit to a blind student's school🧑‍🦯🧑‍🦯(community medicine)
Visit to a blind student's school🧑‍🦯🧑‍🦯(community medicine)lakshayb543
 
Expanded definition: technical and operational
Expanded definition: technical and operationalExpanded definition: technical and operational
Expanded definition: technical and operationalssuser3e220a
 
Grade 9 Quarter 4 Dll Grade 9 Quarter 4 DLL.pdf
Grade 9 Quarter 4 Dll Grade 9 Quarter 4 DLL.pdfGrade 9 Quarter 4 Dll Grade 9 Quarter 4 DLL.pdf
Grade 9 Quarter 4 Dll Grade 9 Quarter 4 DLL.pdfJemuel Francisco
 
Student Profile Sample - We help schools to connect the data they have, with ...
Student Profile Sample - We help schools to connect the data they have, with ...Student Profile Sample - We help schools to connect the data they have, with ...
Student Profile Sample - We help schools to connect the data they have, with ...Seán Kennedy
 
ClimART Action | eTwinning Project
ClimART Action    |    eTwinning ProjectClimART Action    |    eTwinning Project
ClimART Action | eTwinning Projectjordimapav
 
Presentation Activity 2. Unit 3 transv.pptx
Presentation Activity 2. Unit 3 transv.pptxPresentation Activity 2. Unit 3 transv.pptx
Presentation Activity 2. Unit 3 transv.pptxRosabel UA
 
ROLES IN A STAGE PRODUCTION in arts.pptx
ROLES IN A STAGE PRODUCTION in arts.pptxROLES IN A STAGE PRODUCTION in arts.pptx
ROLES IN A STAGE PRODUCTION in arts.pptxVanesaIglesias10
 
THEORIES OF ORGANIZATION-PUBLIC ADMINISTRATION
THEORIES OF ORGANIZATION-PUBLIC ADMINISTRATIONTHEORIES OF ORGANIZATION-PUBLIC ADMINISTRATION
THEORIES OF ORGANIZATION-PUBLIC ADMINISTRATIONHumphrey A Beña
 
Integumentary System SMP B. Pharm Sem I.ppt
Integumentary System SMP B. Pharm Sem I.pptIntegumentary System SMP B. Pharm Sem I.ppt
Integumentary System SMP B. Pharm Sem I.pptshraddhaparab530
 
Oppenheimer Film Discussion for Philosophy and Film
Oppenheimer Film Discussion for Philosophy and FilmOppenheimer Film Discussion for Philosophy and Film
Oppenheimer Film Discussion for Philosophy and FilmStan Meyer
 
Influencing policy (training slides from Fast Track Impact)
Influencing policy (training slides from Fast Track Impact)Influencing policy (training slides from Fast Track Impact)
Influencing policy (training slides from Fast Track Impact)Mark Reed
 
EmpTech Lesson 18 - ICT Project for Website Traffic Statistics and Performanc...
EmpTech Lesson 18 - ICT Project for Website Traffic Statistics and Performanc...EmpTech Lesson 18 - ICT Project for Website Traffic Statistics and Performanc...
EmpTech Lesson 18 - ICT Project for Website Traffic Statistics and Performanc...liera silvan
 

Recently uploaded (20)

YOUVE GOT EMAIL_FINALS_EL_DORADO_2024.pptx
YOUVE GOT EMAIL_FINALS_EL_DORADO_2024.pptxYOUVE GOT EMAIL_FINALS_EL_DORADO_2024.pptx
YOUVE GOT EMAIL_FINALS_EL_DORADO_2024.pptx
 
Measures of Position DECILES for ungrouped data
Measures of Position DECILES for ungrouped dataMeasures of Position DECILES for ungrouped data
Measures of Position DECILES for ungrouped data
 
INCLUSIVE EDUCATION PRACTICES FOR TEACHERS AND TRAINERS.pptx
INCLUSIVE EDUCATION PRACTICES FOR TEACHERS AND TRAINERS.pptxINCLUSIVE EDUCATION PRACTICES FOR TEACHERS AND TRAINERS.pptx
INCLUSIVE EDUCATION PRACTICES FOR TEACHERS AND TRAINERS.pptx
 
4.18.24 Movement Legacies, Reflection, and Review.pptx
4.18.24 Movement Legacies, Reflection, and Review.pptx4.18.24 Movement Legacies, Reflection, and Review.pptx
4.18.24 Movement Legacies, Reflection, and Review.pptx
 
USPS® Forced Meter Migration - How to Know if Your Postage Meter Will Soon be...
USPS® Forced Meter Migration - How to Know if Your Postage Meter Will Soon be...USPS® Forced Meter Migration - How to Know if Your Postage Meter Will Soon be...
USPS® Forced Meter Migration - How to Know if Your Postage Meter Will Soon be...
 
HỌC TỐT TIẾNG ANH 11 THEO CHƯƠNG TRÌNH GLOBAL SUCCESS ĐÁP ÁN CHI TIẾT - CẢ NĂ...
HỌC TỐT TIẾNG ANH 11 THEO CHƯƠNG TRÌNH GLOBAL SUCCESS ĐÁP ÁN CHI TIẾT - CẢ NĂ...HỌC TỐT TIẾNG ANH 11 THEO CHƯƠNG TRÌNH GLOBAL SUCCESS ĐÁP ÁN CHI TIẾT - CẢ NĂ...
HỌC TỐT TIẾNG ANH 11 THEO CHƯƠNG TRÌNH GLOBAL SUCCESS ĐÁP ÁN CHI TIẾT - CẢ NĂ...
 
Keynote by Prof. Wurzer at Nordex about IP-design
Keynote by Prof. Wurzer at Nordex about IP-designKeynote by Prof. Wurzer at Nordex about IP-design
Keynote by Prof. Wurzer at Nordex about IP-design
 
Visit to a blind student's school🧑‍🦯🧑‍🦯(community medicine)
Visit to a blind student's school🧑‍🦯🧑‍🦯(community medicine)Visit to a blind student's school🧑‍🦯🧑‍🦯(community medicine)
Visit to a blind student's school🧑‍🦯🧑‍🦯(community medicine)
 
Expanded definition: technical and operational
Expanded definition: technical and operationalExpanded definition: technical and operational
Expanded definition: technical and operational
 
Grade 9 Quarter 4 Dll Grade 9 Quarter 4 DLL.pdf
Grade 9 Quarter 4 Dll Grade 9 Quarter 4 DLL.pdfGrade 9 Quarter 4 Dll Grade 9 Quarter 4 DLL.pdf
Grade 9 Quarter 4 Dll Grade 9 Quarter 4 DLL.pdf
 
Student Profile Sample - We help schools to connect the data they have, with ...
Student Profile Sample - We help schools to connect the data they have, with ...Student Profile Sample - We help schools to connect the data they have, with ...
Student Profile Sample - We help schools to connect the data they have, with ...
 
ClimART Action | eTwinning Project
ClimART Action    |    eTwinning ProjectClimART Action    |    eTwinning Project
ClimART Action | eTwinning Project
 
Presentation Activity 2. Unit 3 transv.pptx
Presentation Activity 2. Unit 3 transv.pptxPresentation Activity 2. Unit 3 transv.pptx
Presentation Activity 2. Unit 3 transv.pptx
 
LEFT_ON_C'N_ PRELIMS_EL_DORADO_2024.pptx
LEFT_ON_C'N_ PRELIMS_EL_DORADO_2024.pptxLEFT_ON_C'N_ PRELIMS_EL_DORADO_2024.pptx
LEFT_ON_C'N_ PRELIMS_EL_DORADO_2024.pptx
 
ROLES IN A STAGE PRODUCTION in arts.pptx
ROLES IN A STAGE PRODUCTION in arts.pptxROLES IN A STAGE PRODUCTION in arts.pptx
ROLES IN A STAGE PRODUCTION in arts.pptx
 
THEORIES OF ORGANIZATION-PUBLIC ADMINISTRATION
THEORIES OF ORGANIZATION-PUBLIC ADMINISTRATIONTHEORIES OF ORGANIZATION-PUBLIC ADMINISTRATION
THEORIES OF ORGANIZATION-PUBLIC ADMINISTRATION
 
Integumentary System SMP B. Pharm Sem I.ppt
Integumentary System SMP B. Pharm Sem I.pptIntegumentary System SMP B. Pharm Sem I.ppt
Integumentary System SMP B. Pharm Sem I.ppt
 
Oppenheimer Film Discussion for Philosophy and Film
Oppenheimer Film Discussion for Philosophy and FilmOppenheimer Film Discussion for Philosophy and Film
Oppenheimer Film Discussion for Philosophy and Film
 
Influencing policy (training slides from Fast Track Impact)
Influencing policy (training slides from Fast Track Impact)Influencing policy (training slides from Fast Track Impact)
Influencing policy (training slides from Fast Track Impact)
 
EmpTech Lesson 18 - ICT Project for Website Traffic Statistics and Performanc...
EmpTech Lesson 18 - ICT Project for Website Traffic Statistics and Performanc...EmpTech Lesson 18 - ICT Project for Website Traffic Statistics and Performanc...
EmpTech Lesson 18 - ICT Project for Website Traffic Statistics and Performanc...
 

2007

  • 1. 1. A patient is followed up in an revealed localized bright-red hyperemia; endocrinological dispensary on account of tonsils are swollen, soft, lacunas contain pus, hyperthyreosis. Weight loss, tachycardia, finger tongue is crimson. Cervical lymph nodes are tremor are accompanied by hypoxia enlarged, dense and painful. What is the symptoms - headache, fatigue, eye flicker. most probable diagnosis? What mechanism of thyroid hormones action A. Rubella underlies the development of hypoxia? B. Scarlet fever A. Intensification of respiratory ferment C. Infectious mononucleosis synthesis D. Whooping cough B. Inhibition of respiratory ferment synthesis E. Diphtheria C. Specific binding of active centres of 6. A 45 y.o. woman suffers from Cushing's respiratory syndrome - steroid diabetes. Biochemical ferments examination revealed: hyperglycemia, D. Disjunction, oxydation and phosphorilation hypochloremia. Which of the undermentioned E. Competitive inhibition of respiratory processes is the first to be activated? ferments A. Gluconeogenesis 2. A laboratory received a material from B. Glycolysis a patient's wound. Ppreliminary diagnosis C. Glycogenolysis is gaseous gangrene. What microbiological D. Glucose transport to the cell method should be applied to determine E. Glucose reabsorption species of causative agent? 7. Examination of an isolated cardiomyocyte A. Bacteriological revealed that it didn't generate excitation B. Allergic impulses automatically. This cardiomyocyte C. Serological was obtained from: D.RIA A. Sinoatrial node E. Bacterioscopic B. Purkinje's fibers 3: A 4 y.o. child with signs of durative proteinic C. Ventricles starvation was admitted to the hospital. D. Atrioventricular node The signs were as follows: growth inhibition, E. His' bundle anemia, edemata, mental deficiency Choose 8. Inhabitants of territories with cold climate a cause of edemata development: . have high content of an adaptive A. Reduced synthesis of albumins thermoregulatory hormone. What hormone B. Reduced synthesis of hemoglobin is meant? . C. Reduced synthesis of glycoproteins A. Insulin D. Reduced synthesis of globulins B. Glucagon E. Reduced synthesis of lipoproteins C. Throxin 4. A 45 y.o. patient consulted a doctor D. Cortisol about plaque-shaped formation on his neck. E. Somatotropin Histological examination of biopsy skin 9. As a result of damage to certain structures material revealed tumourous cells of round of brainstem an animal lost orientation and oval form with thin ring of basophilic reflexes. What structures were damaged? cytoplasma that resemble of cells of basal A. Medial nuclei of reticular formation epidermal layer. What tumour is it? B. Quadritubercular bodies A. Hydradenoma C. Black substance B. Epidermal cancer D. Reel nuclei C. Trichoepithelioma E. Vestibular nuclei D. Basalioma 10. According to the data of WHO, for about E. Syringoadenoma 250 mln of Earth population fall ill with 5. A boy is 7 y.o. Objectively: against the malaria. This disease is mostly spread in background of hyperemic skin there is tropical and subtropical regions. Range of knobby bright-pink rash on his forehead, its spread falls into the areal of the following neck, at the bottom of abdomen, in the mosquitoes: popliteal spaces; nasolabial triangle is pale. A. Aedes Examination of oropharyngeal surface B. Mansonia
  • 2. C. Culex 16. A patient was diagnosed with active D. Culiseta focal pulmonary tuberculosis. What drug E. Anopheles should be prescribed in the first place? 11. A 35 y.o. patient who often consumes alcohol A. Ethionamide was treated with diuretics. There B. Isoniazid appeared serious muscle and heart C. Ethoxide weakness, vomiting, diarrhea, AP- 100/60 D. Sulfalen mm Hg, depression. This condition is caused E. Cyclocerine by intensified excretion with urine of: 17. Examination of a young man in the AIDS A. Potassium- centre produced a positive result of B. Phosphates immune-enzyme assay with HIV antigens. C. Chlorine Patient's complaints about state of his health D. Sodium were absent. What can the positive result of E. Calcium immune-enzyme assay be evidence of? 12. On the 6th day of treatment a patient A. HIV infection with acute renal insufficiency developed B. Being infected with HBV polyuria. Diuresis intensification at the beginning C. Having had AIDS recently of polyuria stage of acute renal insufficiency D. Being ill with AIDS is caused by: E. HBV persistence A. Growth of natriuretic factor 18. RNA that contains AIDS virus B. Reduction of aldosteron content in plasma penetrated into a leukocyte and by means C. Reduction of vasopressin content in of reverse transcriptase forced a cell to plasma synthetize a viral DNA. This process is D. Renewal of filtration in nephrons based upon: E. Volume expansion of circulating blood A. Operon repression 13. Osmotic pressure of a man's blood B. Operon depression plasma is 350 mosmole/l (standard pressure C. Reverse transcription is 300 mosmole/l). First of all it will result in D. Convariant replication high secretion of the following hormone: E. Reverse translation A. Vasopressin 19. A patient is ill with diabetes mellitus that B. Adrenocorticotropin is accompanied by hyperglycemia of over 7,2 C. Cortisol millimole/l on an empty stomach. The level D. Natriuretic of what blood plasma protein allows to estimate E. Aldosteron the glycemia rate retrospectively (4-8 14. Neurological examination of a 65 y.o. weeks before examination)? patient revealed a haemorrhage within the A. Ceruloplasmin superior temporal gyrus. In the blood supply B. Glycated hemoglobin area of which artery is it? C. Fibrinogen A. Middle cerebral artery D. Albumin B. Anterior communicating artery E. C-reactive protein C. Anterior cerebral artery 20. A sportsman was recommended to take D. Basilar artery a medication that contains carnitine in order E. Posterior cerebral artery to improve his results. What process is activated 15. A 32 y.o. man is tall, he has gynecomastia, by carnitine the most? adult woman pattern of hair distribution, high A. Synthesis of ketone bodies voice, mental deficiency, sterility. Provisional B. Synthesis of steroid hormones diagnosis is Klinefelter's syndrome. In C. Tissue respiration order to specify diagnosis it is necessary to D. Fatty acids transport to mitochondrions analize: E. Synyhesis of lipids A. Leukogram 21. A patient consulted a doctor about B. Genealogy bowels disfunction. The doctor established C. Blood group symptoms of duodenitis and enteritis. D. Spermatogenesis Laboratory examination helped to make the E. Caryotype following diagnosis: lambliosis. What medication
  • 3. should be administered? B. Hyaline degeneration A. Tetracycline C. Carbohydrate degeneration B. Monomycin D. Fatty vascular-stromal degeneration C. Metronidazole E. Amyloidosis D. Erythromycin 27. Labeled aminoacids alanine and E. Chingamin tryptophane were introducted to a mouse 22. A man took a quiet expiration. Name an in order to study localization of protein air volume that is meanwhile contained in biosynthesis in its cells. Around what his lungs: organellas will the accumulation of labeled A. Expiratory reserve volume aminoacids be observed? B. Functional residual capacity A. Ribosomes C. Residual volume B. Golgi apparatus D. Vital lung capacity C. Agranular endoplasmic reticulum E. Respiratory volume D. Cell centre 23. During pubescence the cells of male E. Lysosomes sexual glands begin to produce male 28. 48 hours after performing tuberculin sex hormon testosterone that calls forth test (Mantoux test) to a child a 10 secondary sexual characters. What cells of mm papule appeared on the spot of male sexual glands produce this hormone? tuberculin introduction. What hypersensitivity A. Sertoli's cells mechanism underlies these changes? B. Leidig cells A. Antibody-dependent cytotoxicity C. Sustentocytes B. Granulomatosis D. Spermatozoa C. Cellular cytotoxicity E. Supporting cells D. Immune complex cytotoxicity 24. A 16 y.o. boy from a countryside entered E. Anaphylaxis an educational establishment. Scheduled 29. A patient who has been suffering from Manteux test revealed that the boy had cardiac insufficiency for several months negative reaction. What are the most has been taking digoxin on an outpatient reasonable actions in this case? basis. At a certain stage of treatment there A. To isolate the boy temporarily from his appeared symptoms of drug overdose. What mates phenomenon underlies the development of B. To perform serodiagnostics of tuberculosis this complication? C. To perform BCG vaccination A. Functional cumulation D. To perform rapid Price diagnostics B. Tachyphylaxis E. To repeat the reaction in a month C. Habituation 25. Autopsy of a 56 y.o. man revealed in the D. Sensibilization right temporal part of brain a big focus of E. Material cumulation softened grey matter that was semi-liquid 30. Examination of a patient revealed and light grey. Arteries of cerebral tela reduced contents of magnesium ions that contain multiple whitish-yellow thiekenings are necessary for attachment of ribosomes of intima that abruptly narrow the lumen. to the granular endoplasmatic reticulum. What is your diagnosis? It is known that it causes disturbance of A. Brain edema protein biosynthesis. What stage of protein B. Brain abscess biosynthesis will be disturbed? C. Hemorrhage A. Aminoacid activation D. Hemorrhagic infarction B. Translation E. Ischemic stroke C. Termination 26. Autopsy of a man who died from D. Replication chronic cardiovascular collapse revealed E. Transcription "tiger heart". Sidewards of endocardium 31. A child complains of general weakness, a yellowish-white banding can be seen; loss of appetite, a troubled sleep, itching in myocardium is dull, dark-yellow. What the perianal area. The provisional diagnosis process caused this pathology? is enterobiasis. In order to specify this diagnosis A. Fatty parenchymatous degeneration it is necessary to perform:
  • 4. A. Roentgenoscopy C. Necrotic B. Duodenal contents analysis D. Fibrinous C. Scraping from perianal folds E. Catarrhal D. Immune diagnostics 37. A 70 y.o. man has cut an abscess off E. Biopsy of muscle tissue in the area of mammiform process during 32. In some regions of South Africa there is shaving. Two days later he was admitted a spread sickle-shaped cell anemia, in which to the hospital with inflammation of erythrocytes have shape of a sickle as a arachnoid membranes. How did the infection result of substitution of glutamin by valine in penetrate into the cavity of skull? the hemoglobin molecule. What is the cause A. V,v.auriculares of this disease? B. V.v.labyrinthi A. Genomic mutations C. V.facialis B. Crossingover D. V.v.tympanicae C. Gene mutation E. V.emissariae mastoideae D. Transduction 38. 24 hours after appendectomy blood of a E. Disturbance of mechanisms of genetic patient presents neutrophilic leukocytosis information realization with regenerative shift. What is the 33. A patient suffers from vision impairment most probable mechanism of leukocytosis - hemeralopy (night blindness). What vitamin development? preparation should be administered A. Amplification of leukopoiesis the patient in order to restore his vision? B. Decelerated leukocyte destruction A. Thiamine chloride C. Redistribution of leukocytes in the organism B. Tocopherol acetate D. Deceleratied emigration of leukocytes to C. Retinol acetate the tissues D. Vicasol E. Amplification of leukopoiesis and E. Pyridoxine decelerated emigration of leukocytes to 34. A patient presents high activity of the tissues LDH1.2 , aspartate aminotransferase, creatine 39. Examination of cell culture got from a phosphokinase. In what organ (organs) is patient with lysosomal pathology revealed the development of a pathological process accumulation of great quantity of lipids in the most probable? the lysosomes. What of the following diseases A. In the heart muscle (initial stage of is this disturbance typical for? myocardium infarction) A. Phenylketonuria B. In skeletal muscles (dystrophy, atrophy) B. Galactosemia C. In kidneys and adrenals C. Gout D. In liver and kidneys D. Tay-Sachs disease E. In connective tissue E. Wilson disease 35. A woman who has been keeping to a 40. A group of mountain climbers went clean-rice diet for a long time was diagnosed through the blood analysis at the height of with polyneuritis (beriberi). What vitamin 3000 m. It revealed decrease of HCO3 to 15 deficit results in development of this disease? micromole/l (standard is 22-26 micromole/l). A. Thiamine What is the mechanism of HCO3 decrease? B. Pyridoxine A. Hypoventilation C. Ascorbic acid B. Intensification of acidogenesis D. Folic acid C. Hyperventilation E. Riboflavin D. Decrease of bicarbonate reabsorption in 36. Colonoscopy of a patient ill with kidneys dysentery revealed that mucous membrane E. Decrease of ammoniogenesis of his large intestine is hyperemic, edematic, 41. A patient ill with diabetes mellitus felt its surface was covered with grey-and-green acute pain in his right foot. Objectively: foot coats. Name the morphological form of thumb is black, foot tissues are edematous. dysenteric collitis: there are foci of epidermis desquamation, A. Ulcerous stinking discharges. What clinicopathological B. Purulent form of necrosis is it?
  • 5. A. Infarction C. Decrease of ESR B. Dry gangrene D. Hypercoagulation C. Moist gangrene E. Increase of ESR D. Bedsore 47. Histological examination of a 40 y.o. E. Sequestrum man's thymus revealed decreased share of 42. Power inputs of a boy increased from 500 parenchymatous gland elements, increased to 2000 kJ pro hour. What can be the cause share of adipose and loose connective tissue, of it? its enrichment with thymus bodies. A. Food intake The organ's mass was unchanged. What B. Raise of outer temperatute phenomenon is it? C. Mental activity A. Accidental involution D. Transition from sleep to wakefulness B. Atrophy E. Physical exercise C. Dystrophy 43. For the preparation of a patient`s D. Hypotrophy burn skin surface a certain medication was E. Age involution used. Its antiseptic action is provided by 48. A patient who suffers from cancer of free oxygen that segregates in presence back of tongue has an intense bleeding as a of organic substances. Choose the right result of affection of dorsal lingual artery by answer: the tumour. What vessel should be ligated to A. Potassium permanganate stop bleeding? B. Chlorhexidine A. Facial artery C. Sodium bicarbonate B. Dorsal lingual artery D. Boric acid C. Ascending pharyngeal artery E. Furacilin D. Lingual artery 44. Bacteriological examination of a patient E. Deep lingual artery with food poisoning required inoculation of 49. In course of indirect histogenesis of a pure culture of bacteria with the following tubular bone tissue a plate is formed properties: gram-negative movable bacillus between epiphyseal and diaphyseal ossification that grows in the Endo's medium in form of centres that provides further colourless colonies. A representative of which lengthwise growth of bones. What structure species caused this disease? is it? A. Esherichia A. Osseous plate B. Salmonella B. Metaphyseal plate C. Shigella C. Osteon D. Citrobacter D. Osseous cuff E. Iersinia E. Layer of interior general plates 45. A lymph node punctate of a patient with 50. A man with cut wound of his right foot suspected protozoal disease was examined. sole was admitted to the hospital ward. The Examination of the stained specimen patient has limited elevation of the lateral (Romanovsky's stain) revealed some foot edge. In course of wound management crescent bodies with pointed end, blue the injury of a muscle tendon was revealed. cytoplasm and red nucleus. What protozoan What muscle is injured? were revealed in the smears? A. Triceps muscle of crus A. Viscerotropic leishmania B. Long extensor muscle of toes B. Toxoplasms C. Long peroneal C. Trypanosomes D. Short peroneal D. Malarial plasmodiums E. Anterior tibial E. Demtropic leishmania 51. After resection of the middle third of 46. Long-term starvation cure of a patient femoral artery obliterated by a thromb the resulted in diminished ratio of albumines lower extremity is supplied with blood due and globulines in plasma. What of the to the surgical bypass. Name an artery that following will be result of these changes? plays the main role in reestablishment of A. Decrease of hematocrit blood flow: B. Increase of hematocrit A. Deep femoral artery
  • 6. B. Deep external pudendal artery branches of carotid artery. For a temporary arrest C. Descending genicular artery of bleeding it is necessary to press the D. Superficial circumflex artery of hip bone carotid artery to the tubercle of a cervical E. Superficial epigastric artery vertebra. Which vertebra is it? 52. An experimental animal was first sensibilized A.VI whereupon an antigen dose was B. II introduced subcutaneously. This injection C. IV resulted in the development of a fibrinous D. III inflammation with alteration of vessel E. V walls, basal substance and fibrous structures 57. Reaction of passive hemagglutination of connective tissue in form of mucoid conducted with erythrocytic typhoid and fibrinoid swelling and necrosis. What Vidiagnosticum immunological reaction took place? helped to reveal some antibodies A. Immediate hypersensitivity in the dilution of the patient's serum B. Granulomatosis at a ratio of 1:80 that exceeds the diagnostic C. Reaction of transplantation immunity titer. Such result witnesses of: D. Delayed-type hypersensitivity A. Typhoid fever recurrence E. Normergic reaction B. Being a potential carrier of typhoid bacilli 53. Autopsy of a patient who suffered C. Being ill with acute typhoid fever from croupous pneumonia and died from D. Incubation period of typhoid fever pneumococcal sepsis revealed 900 ml of E. Reconvalescence of a patient ill with turbid greenish-yellow liquid in the right typhoid fever pleural cavity. Pleural leaves are dull, 58. A patient with clinical presentations plephoric. Name the clinicopathological of immunodeficiency went through form of inflammation in the pleural cavity: immunological examinations. They revealed A. Acute abscess significant loss of cells that form rosettes with B. Empyema erythrocytes of a ram. What conclusion C. Phlegmon can be made according to the analysis data? D. Fibrinous inflammation A. Decrease of natural killer cell rate E. Chronic abscess B. Decrease of T-lymphocytes rate 54. A patient was delivered to the hospital C. Insufficiency of effector cells of humoral by an emergency team. Objectively: immunity grave condition, unconscious, adynamy. D. Decrease of B-lymphocytes rate Cutaneous surfaces are dry, eyes are sunken, E. Decrease of complement system rate face is cyanotic. There is tachycardia and 59. Examination of a 60 y.o. patient revealed smell of acetone from the mouth. Analysis hyperglycemia and glucosuria. A doctor results: blood glucose - 20,1 micromole/l administered him a medication for internal (standard is 3,3-5,5 micromole/l), urine use. What medication is it? glucose - 3,5% (standard is - 0). What is the A. Oxytocin most probable diagnosis? B. Corglycon A. Hypoglycemic coma C. Furosemide B. Acute heart failure D. Pancreatine C. Anaphylactic shock E. Glibenclamid D. Hyperglycemic coma 60. In order to estimate toxigenity of diphtheria E. Acute alcoholic intoxication agents obtained from patients the 55. A patient has a disturbed absorbtion of cultures were inoculated on Petri dish with fat hydrolysates. It might have been caused nutrient agar on either side of a filter paper by a deficit in the small intestine cavity: strip that was put into the centre and moistened A. Of bile pigments with antidiphtherie antitoxic serum. B. Of sodium ions After incubation of inoculations in agar the C. Of bile acids strip-like areas of medium turbidity were D. Of lipolytic enzymes found between separate cultures and the E. Of liposoluble vitamins strip of filter paper. What immunological 56. An injured man has bleeding from reaction was conducted?
  • 7. A. Opsonization reaction E. 70 B. Coomb's test 66. A 40 y.o. patient complains of intensive C. Agglutination reaction heartbeats, sweating, nausea, vision impairment, D. Rings precipitation reaction arm tremor, hypertension. From his E. Precipitation gel reaction anamnesis: 2 years ago he was diagnosed with 61. A woman with III (B), Rhˉ blood group pheochromocytoma. Hyperprocduction of born a child with II (A) blood group. The what hormones causes the given pathology? child is diagnosed with hemolytic disease of A. ACTH newborn as a result of rhesus incompatibility. B. Thyroid hormones What blood group is the child's father C. Aldosterone likely to have? D. Glucocorticoids A. I (0). Rh + E. Catecholamines B. III (B). Rh+ 67.Mucous membrane of the right palatine tonsil C. I (0), Rh- has a painless ulcer with smooth lacquer fundus D. II (A), Rh- and regular cartilaginous edges. Microscopically: E. II (A), Rh+ inflammatory infiltration that consists of 62. A 12 y.o. boy who suffers from bronchial lymphocytes, plasmocytes, a small number of asthma has an acute attack of asthma: evident neutrophils and epithelioid cells; endovasculitis expiratory dyspnea, skin pallor. What and perivasculitis. What disease is it? type of alveolar ventilation disturbance is it? A. Actinomycosis A. Central B. Pharyngeal diphtheria B. Neuromuscular C. Ulcerous necrotic Vincent’s angina C. Throracodiaphragmatic D. Tuberculosis D. Restrictive E. Syphilis E. Obstructive 68. For the purpose of retrospective diagnostics of 63. A patient diagnosed with carcinoid recent bacterial dysentery it was decided to of bowels, was admitted to the hospital. perform serological examination of blood serum Analysis revealed high production of in order to determine antibody titer towards Shiga serotonin. It is known that this substance bacilli. What of the following reactions should be is formed of tryptophane aminoacid. applied? What biochemical mechanism underlies this A. Passive hemagglutination process? B. Bordet-Gengou test A. Microsomal oxydation C.Hemolysis B. Formation of paired compounds D. Precipitation C. Desamination E. Bacteriolysis D. Decarboxylation 69. Autopsy of a 48 y.o. man revealed a round E. Transamination formation 5 cm in diameter with clear-cut 64. Examination of a patient revealed an outlines in the region of the 1st segment of his abscess of pterygopalatine fossa. Where can right lung. This formation was encircled with a the infection spread to unless the disease is thin layer of connective tissue full of white brittle managed in time? masses. Make a diagnosis of the secondary A. To the orbit tuberculosis form: B. To the subgaleal temporal space A. Fibrous cavernous tuberculosis C. To the tympanic cavity B. Tuberculoma D. To the interpterygoid space C. Acute focal tuberculosis E. To the frontal sinus D. Caseous pneumonia. 65. Examination of a man established that E. Acute cavernous tuberculosis cardiac output equaled 3500 ml, systolic 70. A man’s intrapleural pressure is being output - 50 ml. What is the man's heart rate measured. In what phase did the man hold his pro minute? breath, if his pressure is 7,5 cm Hg? A.60 A. Forced expiration B.90 B. Quiet inspiration C. 50 C. – D. 80 D. Quiet expiration
  • 8. E. Forced inspiration B. Fibrinous inflammation 71. In the surgical department of a hospital there C. Hyalinosis was an outbreak of hospital infection that showed D. Mucoid swelling itself in often postoperative wound abscesses. E. Amyloidosis Bacteriological examination on of pus revealed 76. In course of an operation surgeon aurococcus. What examination shall be conducted removed a part of a lung that was ventilated to find out the source of this causative agent by a tertiary bronchus accompanied among the department personnel? by branches of pulmonary artery and other A. Serological identification vessels. What part of a lung was removed? B. Microscopical examination A. Middle lobe C. Phagotyping B. Bronchopulmonary segment D. Biochemical identification C. Superior lobe E. Estimation of antibiotic susceptibility D. Inferior lobe 72. Heart rate of a man permanently E. Pulmonary lobule equals 40 beats pro minute. What is the 77. Vitamin A deficit results in the impairment pacemaker? of twilight vision. Name the cells that A. His' bundle have the above-mentioned photoreceptor B. Purkinje's fibers function: C. His' bundle branches A. Ganglion neurocytes D. Sinoatrial node B. Rod receptor cell E. Atriventricular node C. Cone receptor cells 73. A 4 y.o. boy has had recently serious D. Horizontal neurocytes viral hepatitis, Now there are such E. Bipolar neurons clinical presentations as vomiting, loss of 78. Microscopical examination of a removed consciousness, convulsions. Blood analysis appendix revealed an edema, diffuse revealed hyperammoniemia. Disturbunce neutrophilic infiltration of appendix wall of which biochemical process caused such along with necrosis and defect of mucous pathological condition of the patient? membrane with affection of its muscle plate. A. Disturbed neutralization of ammonia in What appendicitis form was developed? liver A. Gangrenous B. Inhibition of transamination enzyms B. Ulcerophlegmonous C. Increased putrefaction of proteins in C. Apostematous bowels D. Superficial D. Activation of aminoacid decarboxylation E. Phlegmonous E. Disturbed neutralization of biogenic 79. Introduction of a pharmaceutical amines substance to an experimental animal 74. An experimental rat with extremity resulted in reduction of salivation, pupil paralysis has no tendon and cutaneous mydriasis. Next intravenous introduction of reflexes, muscle tone is decreased, but acetylcholine didn't lead to any significant muscles of the affected extremity maintain changes of heart rate. Name this substance: their ability to react with excitation to the direct A. Proserin action of continious current. What type B. Atropine of paralysis is it? C. Propranolol A. Spastic peripheral D. Adrenaline B. Spastic central E. Salbutamol C. Extrapyramidal 80. Examination of a 43 y.o, anephric patient D. Flaccid peripheral revealed anemia symptoms. What is the E. Flaccid central cause of these symptoms? 75. Autopsy of a 58 y.o, man revealed that A. Folic acid deficit bicuspid valve was deformed, thickened and B. Iron deficit unclosed. Microscopically: foci of collagen C. Reduced synthesis of erythropoietins fibrilla are eosinophilic, react positively to D. Vitamin B12 deficit fibrin. The most probably it is: E. Enhanced destruction of erythrocytes A. Fibrinoid swelling 81. Examination of a newborn boy's genitals
  • 9. revealed a cleft of urethra that opens retrosternal pain a patient presented a jump on the inferior surface of his penis. What of aspartate aminotransferase activity in developmental anomaly is it? blood serum. What pathology is this deviation A. Cryptorchism typical for? B. Epispadia A. Myocardium infarction C. Monorchism B. Viral hepatitis D. Hennaphroditism C. Diabetes mellitus E. Hypospadia D. Collagenosis 82. A patient ill with collagenesis has E. Diabetes insipidus been taking prednisolone for a long time. 87. A peripheral segment of vagus nerve on Hypokaliemia development caused spastic pain of a dog's neck was being stimulated in course skeletal muscles. What medication of an experiment. The following changes should be used in order to correct potassium of cardiac activity could be meanwhile exchange? observed: A. Dithylinum A. Heart rate and heart force amplification B. Panallgin B. Enhancement of atrioventricular C. Thyrocalcitonin conduction D. Diazepam C. Heart rate fall E. Noshpa D. Heart hurry 83. From pharynx of a child with suspected E. Increased excitability of myocardium diphtheria a pure culture of microorganisms 88. A 39 y.o. woman went through an was isolated. Their morphological tinctorial, operation in course of which surgeons cultural and biochemical properties removed her uterine tube that was enlarged appeared to be typical for diphtheria and a part of an ovary with a big cyst. causative agents. What study should be Histological examination of a tube wall conducted in order to drow a conclusion that revealed decidual cells, chorion villi. What this is a pathogenic diphtheria bacillus? was the most probable diagnosis made after A. Estimation of proteolytic properties examination of the uterine tube? B. Estimation of urease activity A. Tubal pregnancy C. Estimation of cystinous activity B. Placental polyp D. Estimation of ability to decompose starch C. Papyraceous fetus E. Estimation of toxigenic properties D. Lithopedion 84. A man was admitted to the hospital E. Choriocarcinoma on the 5th day of disease that manifested 89. A forensic medical expert examines itself by jaundice, muscle aching, chill, nose the body of a 58 y.o. man who had been bleedings. In course of laboratory diagnostics consuming large amounts of alcochol for a a bacteriologist performed dark-field microscopy long time and died at home. Microscopicaly: of the patient's blood drop. Name the right lung is dense and enlarged, its incision a causative agent of this disease: revealed that the tissue is greyish and A. Leptospira interrogans homogenous, pleura is covered with greyish B. Bartinella bacilloformis layers. Microscopically - alveolar cavities C. Calymmatobacterium granulomatis contain fibrin, hemolyzed erythrocytes. D. Rickettsia mooseri Make a diagnosis: E. Borrelia dutlonii A. Focal pneumonia 85. Nappies of a newborn have dark spots B. Caseous pneumonia that witness of formation of homogentisic C. Interstitial pneumonia acid. Metabolic imbalance of which D. Primary pulmonary tuberculosis substance is it connected with? E. Croupous pneumonia A. Methionine 90. A 40 y.o. woman was admitted to the B. Thyrosine infectious diseases department with high C. Galactose body temperature. Objectively: evident D. Tryptophane meningeal symptoms. A spinal cord punction E. Cholesterine was made. What anatomic formation was 86. 12 hours after an accute attack of punctured?
  • 10. A. Cisterna cerebellomedullaris posterior tissue will be inhibited? B. Spatium subdurale A. Of fibroblasts C. Spatium epidurale B. Of pericytes D. Spatium subarachnoideum C. Of pigment cells E. Cavum trigeminale D. Of lipocytes 91. A patient with II stage hypertension has E. Of macrophags been taking one of hypotensive medications 96. 6 months after delivery a woman had for the purpose of treatment. After a time uterine bleeding. Gynecological examination arterial pressure decreased, but the patient revealed in the uterine cavity a dark-red started complaining of flaccidity, sleepiness, tissue with multiple cavities that resembled indifference. A bit later he felt stomach of "sponge". Microscopic examination of pain. He was diagnosed with ulcer. What the tumour revealed some atypic light epithelial hypotensive medication has the patient been Langhans cells and giant cells of taking? cyncytiotrophoblast in blood lacunas. What A. Captopril tumour is it? B. Dibazoie A. Vesicular mole C. Reserpine B. Fibromyoma D. Furosemide C. Adenocarcinoma E. Verapamil D. Squamous cell nonkcratinous carcinoma 92. A patient had to go through an operation. E. Chorioepithelioma Doctors introduced him dithylinum 97. A 50 y.o. patient with chronic cardiac (listenone) and performed intubation. insufficiency and tachyarrythmia was After the end of operation and cessation prescribed a cardiotonic drug. What drug of anesthesia the independent respiration wasn't was prescribed? restored. Which enzyme deficit A. Dobutamine prolongs the action of muscle relaxant? B. Mildronate A. Pseudocholinesterase C. Amyodarone B. K-Na-adenosine triphosphatase D. Digoxin C. N - acetyltransferase E. Dopamine D. Succinate dehydrogenase 98. A young man consulted a doctor about E. Carbanhydrase disturbed urination. Examination of his 93. Ammonia is a very toxic substance, external genitals revealed that urethra is especially for nervous system. What split on top and urine runs out of this substance takes the most active part 111 opening. What anomaly of external genitals ammonia detoxication in brain tissues? development is the case? A. Glutamic acid A. Paraphimosis B. Alanine B. Epispadia C. Proline C. Hypospadia D. Histidine D. Hermaphroditism E. Lysine E. Phimosis 94. A child's blood presents high content 99. Violation of safety rules resulted in of galactose, glucose concentration is low. calomel intoxication. Two days later the There are such presentations as cataract, daily diuresis was 620 m1. A patient mental deficiency, adipose degeneration of experienced headache, vomiting, convulsions, liver. What disease is it? dyspnea, moist rales in lungs. What A. Galactosemia pathology is it? B. Steroid diabetes A. Acute renal insufficiency C. Lactosemia B. Chronic renal insufficiency D. Diabetes mellitus C. Uraemic coma E. Fructosemia D. Pyelonephritis 95. In course of an experiment a big number E. Glomerulonephritis of column cells of red bone marrow was in 100. A patient has a transverse disruption of some way destructed. Regeneration of which cell spinal cord below the IV thoracic segment. populations in the loose connective What changes of respiration will it cause?
  • 11. A. Respiration will become more frequent C. Melancholic B. Respiration will stay unchanged D. Sanguine C. Respiration will stop E. - D. Respiration will become less frequent 106. A patient suffers from hepatic cirrhosis. E. Respiration will become deeper Examination of which of the following 101. A patient with a stab wound of the substances excreted by urine can characterize anterior stomach wall is in surgical care. the state of antitoxic function of liver? What formation of abdominal cavity did the A. Hippuric acid stomach contents get into? B. Ammonium salts A. Right mesenteric sinus C. Aminoacids B. Hepatic bursa C. Omental bursa D. Kreatinine D. Left mesenteric sinus E. Uric acid E. Antegastrial bursa 107. Examination of a child revealed some 102. A patient with infectious mononucleosis whitish spots looking like coagulated milk has been taking glucocorticoids for two on the mucous membrane of his cheeks and weeks. He was brought into remission, but tongue. Analysis of smears revealed grampositive he fell ill with acute attack of chronic tonsillitis. oval yeast-like cells. What causative What action of glucocorticoids caused agents are they? this complication? A. Actinomycetes A. Anti-inflammatory B. Staphylococci B. Immunosuppressive C. Candida C. Antiallergic D. Fusobacteria D. Antishock E. Diphtheria bacillus E. Antitoxic 108. A patient's knee joint doesn't extend, 103. Removal of gall bladder of a patient there is no knee-jerk ref1ex, skin sensitivity has disturbed processes of Co absorption of the anterior femoral surface is disturbed. through the intestinal wall. What vitamin What nerve structures are damaged? will stimulate this process? A. Superior gluteal nerve A. B12 B. Femoral nerve B. PP C. Big fibular nerve C. C D. Inferior gluteal nerve D. K E. Obturator nerve E. D3 109. A patient has pain, edema and reddening 104. A 9 m.o. child has delayed dentition, it of his skin in the anterosuperior area of is also out of order. Upper jaw configuration his thigh and his foot's thumb. What lymph is horizontal ("high palate); microscopically nodes of his lower extremity responded to - irregular mineralization of tooth enamel, the inflammatory process? wrinkled enamel prisms, some of them are A. Deep inguinal vacuolized. Predentin zone is extended; B. Superficial longitudinal there are solitary denticles. What disease C. Superficial inguinal is it? D. lnternal longitudinal A. Late rickets E. General longitudinal B. Early rickets 110. Examination of a 43 y.o. patient C. Hypervitaminosis D revealed that his stomach has difficulties with D. Gout digestion of protein food. Gastric juice E. Osteon1alacia analysis revealed low acidity. Function of 105. Examination of a patient revealed which gastric cells is disturbed in this case? a strong, balanced, inert type of higher A. Main exocrinocytes nervous activity according to Pavlov. What B. Parietal exocrinocytes temperament type does the patient have C. Mucous cells (mucocytes) (according to Hiprocrates classification)? D. Endocrinous cells A. Phlegmatic E. Cervical mucocytes R Choleric 111. The permeability of the irritable cell
  • 12. membrane has been increased for potassium C. Lactin and myosin synthesis ions during an experiment. What changes of D. Regulation of Ca2+ rate in mitochondrions membrane electric status can occur? E. Substrate phosphorylation A. Action potential 117. A 30 y.o. woman had been ill for a B. No changes year when she felt pain in the area of joints C. Hyperpolarization for the first time, they got swollen and D. Local response skin above them became reddened. Provisional E. Depolarization diagnosis is rheumatoid arthritis. One 112. An ovary specimen stained by of the most probable causes of this disease is hematoxylin-eosin presents a follicle, where a structure alteration of a connective tissue cells of follicular epithelium are placed in protein: 1-2 layers and have cubic form, there is a A. Collagen bright-red membrane around the ovocyte. B. Troponin What follicle is it? C. Ovoalbumin A. Secondary D. Mucin B. Atretic E. Myosin C. Primordial 118. A 56 y.o. patient has been suffering from D. Primary thyreotoxicosis for a long time. What type of E. Mature hypoxia can be developed? 113. Utilization of arachidonic acid via A. Tissue cyclooxigenase pathway results in formation B. Hemic of some bioactive substances. Name them: C. Circulatory A. Prostaglandins D. Respiratory B. Biogenic amins E. Mixed C. Insulin-like growth factors 119. A 48 y.o. patient was admitted to the D. Somatomedins hospital with complaints about weakness, E. Thyroxine irritability, sleep disturbance. Objectively: 114. A woman has been applying a new skin and scleras are yellow. In blood: cosmetic preparation for a week that conjugated bilirubin, cholalemia. Feces are resulted in eye-lid inflammation accompanied acholic. Urine is of dark colour (bilirubin). by hyperemia, infiltration and painfulness. What jaundice is it? What type of allergic reaction was A. Mechanic developed? B. Crigler-Najjar syndrome A.IV C. Hemolytic B. I D. Parenchymatous C. III E. Gilbert's syndrome D.V 120. A lightly dressed man is standing in a E. II room, air temperature is +14°C, windows 115. A doctor administered a patient with and doors are closed. In what way does he allergic dermatitis a H1 -histamine blocker emit heat the most actively? as a part of complex treatment. Name this A. Perspiration medication: B. Heat conduction A. Prednisolone C. Heat radiation B. Loratadine D. Convection C. Cromolyn sodium E. Evaporation D. Hydrocortisone 121. A patient complains of pain in the area E. Adrenaline of his liver. Duodenal intubation revealed 116. A 1 y.o. child with symptoms of muscle yellowish, oval, narrowed at the poles eggs affection was admitted to the hospital. with an operculum at the end. Size of these Examination revealed carnitine deficit in eggs is the smallest among all helminth eggs. muscles. Biochemical base of this pathology What is the most probable diagnosis? is disturbed process of: A. Beef tapeworm infection A. Transporting of fatty acids to mitochondrions B. Diphyllobothriasis B. Lactic acid utilization C. Echinococcosis
  • 13. D. Teniasis a worker has largely reduced buffer capacity E. Opisthorchosis of blood. What acidic substance that came 122. A patient who suffers from severe disorder to blood caused this phenomenon? of water-salt metabolism experienced A. - cardiac arrest in diastole. What is the most B. l,3-bisphosphoglycerate probable mechanism of cardiac arrest in diastole? C. 3-phosphoglycerate A. Hypernatremia D. Pyruvate B. Hyponatremia E. Lactate C. Hypokaliemia 128. Bacteriological laboratory examines D. Organism dehydratation canned meat whether it contains botulinum E. Hyperkaliemia toxin. For this purpose an extract of test 123. A 36 y.o. man has a craniocerebral specimen and antitoxic antibotulinic scrum trauma. Objectively: diminished breath of A, B, E types were introducted to a group sounds, thready pulse, no reflexes. What way of mice under examination; a control group of pyracetam introduction will be the most of mice got the extract without antibotulinic apropriate in this case? serum. What serological reaction was applied? A. Subcutaneous A. Complement binding B. Intravenous B. Precipitation C. Inhalation C. Opsono-phagocytic D. Rectal D. Neutralization E. Peroral E. Double immune diffusion 124. Short-term physical activity resulted in 129. Parents of a 10 y.o. boy consulted a reflex amplification of heart rate and raise of doctor about extension of hair-covering, systemic arterial pressure. What receptors growth of beard and moustache, low voice. activation was the main cause of pressor Intensified secretion of which hormone reflex realization? must be assumed? A. Vascular chemoreceptors A. Of testosterone B. Vascular volume receptors B. Of somatotropin C. Proprioreceptors of active muscles C. Of cortisol D. Hypothalamus thermoreceptors D. Of oestrogen E. Vascular baroccptors E. Of progesterone 125. A patient with neuritis of femoral nerve 130. A 60 y.o. patient has a reduced has disturbed flexion of thigh as well as disturbed perception of high-frequency sounds. What crus extension in the knee joint. structures' disorder of auditory analizer What muscle's function is disturbed? caused these changes? A. Quadriceps muscle of thigh A. Main membrane of cochlea near helicotrema B. Semitendinous muscle B. Main membrane of cochlea near the oval C. Triceps muscle of thigh window D. Biceps muscle of thigh C. Muscles of middle ear E. Semimembranous muscle D. Eustachian tube 126. A 55 y.o. woman consulted a doctor about E. Tympanic membrane having continuous cyclic uterine 131. In course of an experiment a skeletal hemorrhages for a year, weakness, dizziness. muscle is being stimulated by a series of Examination revealed skin pallor. electric impulses. What type of muscle Hcmogram: Hb- 70 g/L erythrocytes 3.2*1012/l, contraction will arise, if every subsequent color index - 0,6, leukocytcs 6.0*109/l, impulse comes in the period of relaxation of reticulocytes - 1%; erythrocyte single muscle contraction? hypochromia. What anemia is it? A. Muscle contructure A. Hemolytic anemia B. A series of single contractions B. B12 -folate-deficiency anemia C. Holotetanus C. Chronic posthemorrhagic anemia D. Asynchronous tetanus D. Iron-deficiency anemia E. Partial tetanus E. Aplastic anemia 132. A liquidator of a breakdown at a 127. As a result of exhausting muscular work nuclear power plant who was irradiated
  • 14. complained about vomiting that occurs all E. Lisosomes of a sudden. What medication should be 137. A 22 y.o. woman has enlarged lymph prescribed? nodes. Histologlcally: a lymph node contains A. Metoclopramide lymphocytes, histiocytes, reticular cells, B. Aeron small and big Hodgkin's cells, multinucleated C. Reserpine Sternberg cells, isolated foci of D. De-Nol caseous necrosis. What disease are these E. Atropine changes typical for? 133. A patient got a craniocerebral trauma A. Lung cancer metastasis that resulted in right-side convergent strabismus. B. Chronic leukosis Damage of which craniocerebral C. Acute leukosis nerve caused such consequences? D. Lymphogranulomatosis A. n.facialis E. Lymphosarcoma B. n.trigeminus 138. A 49 y.o. woman consulted a doctor C. n.aculomotorius about heightened fatigue and dyspnea during D. n.trochlearis physical activity. ECG: heart rate is E. n.abducens 50/min, PQ is extended, QRS is unchanged, 134. A patient ill with bronchial asthma didn't P wave quanity exceeds quantity of QRS inform his doctor that he had attacks complexes. What type of arrhythmia does of stenocardia. Doctor administered him a the patient have? medication, which taking resulted in less A. Extrasystole frequent attacks of bronchial asthma. but B. Sinoatrial block stenocardia attacks became more frequent. C. Sinus bradycardia What medication was administered? D. Atrioventricular block A. Cromolyn sodium E. Ciliary arhythmia B. Salbutamol 139. A patient who suffers from pneumonia C. Aminophylline has high body temperature. What biologically D. Phenotherol active substance plays the leading part E. Isadrin in origin of this phenomenon? 135. Autopsy of a man who died from A. Bradykinin influenza revealed that his heart was slightly B. Leukotrienes enlarged, pastous, myocardium was C. Histamine dull and had specks. Microscopical cxamination D. Serotonin of myocardium revealed signs of E. Interleukin-I parenchymatous adipose and hydropic 140. A teenager was irradiated with high dystrophy; stroma was edematic with poor radiation dose that resulted in serious macrophagal and lymphocytic infiltration, damages of lymphoid system, lysis of vessels were plethoric; perivascular analysis many lymphocytes. Restoration of normal revealed petechial hemorrhages. What type hemogram is possible due to the functioning of myocarditis was developed in this case? of the following gland: A. Serous diffuse A. Thyroid B. Granulomatous B. Adrenal C. Purulent C. Liver D. Interstitial proliferative D. Thymus E. Serous focal E. Pancreas 136. In course of practical training students 141. In course of histidine catabolism a biogenic studied a stained blood smear of a mouse with amin is formed that has powerful bacteria phagocyted by leukocytes. What vasodilatating effect. Name it: cell organella completes digestion of these A. Histamine bacteria? B. Dopamine A. Mytochondrions C. Dioxyphenylalanine B. Ribosomes D. Noradrenalin C. Golgi apparatus E. Serotonin D. Granular endoplasmic reticulum 142. According to audiometry data a patient
  • 15. has a disturbed perception of mediumfrequency A. Spinothalamic sounds. It might have been caused B. Medial spinocortical by a damage of: C Anterior spinocerebellar A. Cochlear nuclei D. Posterior spinocerebellar B. Quadritubercular structure E. Lateral spinocortical C. Middle part of helix 147. A patient who suffers from heart failure D. Lateral geniculate bodies has enlarged liver, edemata of lower E. Spiral ganglion extremities, ascites. What is the leading 143. A patient with fracture of his lower mechanism in the development of this jaw was admitted to the maxillofacial edema? department. It was decided to fix his A. Hydrodynamic bones surgically under anaesthetic. After B. Membranogenic intravenous introduction of muscle relaxant C- there arose short fibrillar contractions of D. Lymphogenous the patient's facial muscles. What muscle E. Colloid osmotic relaxant was applied? 148. A rabbit's nerve that innervates the A. Dithylinum right ear was cut and its right superior B. Pipecuronium bromide cervical ganglion was removed. Immediately C. Diazepam after operation the temperature of D. Melictine ear skin was measured. It was revealed that E. Tubocurarin chloride the temperature of the rabbit's ear skin on 144. Analysis of blood serum of a patient the side of denervation was by 1,5°C higher revealed increase of alanine than on the opposite intact side. What of the aminotransferase and aspartate aminotransferase following is the most probable explanation level. What cytological of the above-mentioned effects? changes can cause such a situation? A. Atrerial hyperemia induced by metabolic A. Disturbance of cellular interrelations factors B. Cellular breakdown B. Arterial neuroparalytic hyperemia C Disturbance of genetic apparatus of cells C. Arterial neurotopical hyperemia D. Disturbed function of energy supply of D. Reactive arterial hyperemia cells E. Physiological arterial hyperemia E. Disorder of enzyme systems of cells 149. Inflammation of a patient's eye was 145. A 63 y.o. man fell ill with acute tracheitis accompanied by accumulation of turbid liquid and bronchitis accompanied by bronchial with high protein at the bottom of pneumonia. On the l0th day the patient anterior chamber that was called hypopyon. died from cardiopulmonary insufficiency What process underlies the changes under Autopsy revealed fibrinous hemorrhagic observation? laryngotracheobronchitis: lungs were A. Primary alteration enlarged. their incision revealed-the "coalminer's" B. Proliferation effect caused by interlacing of sections C.- of bronchial pneumonia. hemorrhages D. Secondary alteration into the pulmonary parenchyma, acute E. Disturbance of microcirculation abscesses and atelectases. Internal organs 150. A patient was ill with burn disease that have discirculatory and dystrophic changes. was complicated by DIC syndrome. What What is the most probable diagnosis? stage of DIC syndrome can he suspected if it A. Moderately severe influenza IS known that the patient's blood coagulates B. Adenoviral infection in less than 3 minutes? C Parainfluenza A. Terminal D. Influenza. severe form B. Transition phase E. Respiratory syncytial infection C. Hypocoagulation 146. As a result of spinal-cord trauma D. Hypercoagulation a 33 y.o. man has a disturbed pain and E. Fibrinolysis temperature sensitivity that is caused by 151. An experimental animal has been given damage of the following tract: excessive amount of carbon-Iabeled glucose
  • 16. for a week. What compound can the label B. Supracutaneous tularin test be found in? C. Mantoux test A. Methionine D. Burnet test B. Arachidonic acid E. Anthraxine test C. Palmitic acid 157. A patient has extrasystole. ECG shows D. Choline no P wave, QRS complex is deformed, E. Vitamin A there is a full compensatory pause. What 152. A patient complains of dryness of extrasystoles are these? head skin, itching, fragility and loss of hair. A. Atrioventricular After examination he was diagnosed with B. Sinus seborrhea. Disturbed activity of which cells C. Ventricular caused this condition? D. Atrial A. Cells of sudoriferous glands E. - B. Melanocytes 158. After intake of rich food a patient C. Adipocytes feels nausea and sluggishness; with time D. Epithelial cells there appeared signs of steatorrhea. E. Cells of sebaceous glands Blood cholesterine concentration is 9,2 153. A patient who had been working hard micromole/l. This condition was caused by lack under conditions of elevated temperature of: of the environment, has now a changed A. Fatty acids quantity of blood plasma proteins. What B. Bile acids penomenon is the case? C. Phospholipids A. Paraproteinemia D. Chylomicrons B. Absolute hyperproteinemia E. Triglycerides C. Disproteinemia 159. A 27 y.o. patient put eye drops D. Relative hyperproteinemia that contain penicillin. After a few minutes E. Absolute hypoproteinemia she felt itching and burning of 154. Examination of a patient revealed her body, there appeared lip and eye-lid extremely myotic pupils, sleepiness, edemata; arterial pressure began to drop. infrequent Chain-Stoke's respiration. urinary What immunoglobulins took part in the retention, slowing-down of heart development of this allergic reaction? rate, enhancement of spinal reflexes. What A. IgE and IgG substance caused the poisoning? B. IgG and IgD A. Atropine C. IgM and IgD B. Phosphacole D. IgM and IgG C. Caffeine E. IgA and IgM D. Morphine 160. A patient suffers from severe E. Barbital postoperative pseudomonadous infection. 155. Examination of a patient with frequent What of the following antibiotics should be hemorrhages from internals and mucous administered in this case? membranes revealed proline and lysine A. Erythromycin being a part of collagene fibers. What vitamin B. Doxycycline absence caused disturbance of their C. Cephazolin hydroxylation? D. Benzylpenicillin A. Vitamin K E. Amicacin sulfate B. Vitamin E 161. A patient complains of frequent C. Vitamin A diarrheas, especially after consumption D. Vitamin e of fattening food, and of body weight E. Thiamine loss. Laboratory examination revealed 156. The first grade pupils were examined in steatorrhea: hypocholic feces. What can be order to sort out children for tuberculosis the cause of this condition? revaccination. What test was applied for this A. Mucous membrane inflammation of small purpose? intestine A. Schick test B. Unbalanced diet
  • 17. C. Lack of pancreatic phospholipase C. Manitole D. Obturation of biliary tracts D. Clopamide E. Lack of pancreatic lipase E. Amyloride 162. Having helped to eliminate 167. A newborn child with pylorostenosis consequences of a failure at a nuclear has often repeating vomiting accompanied power plant, a worker got an irradiation by apathy, weakness, hypertonicity, sometimes doze of 500 roentgen. He complains of convulsions. What disorder form of headache, nausea, dizziness. What changes acid-base balance is it? in leukocytes quantity can be expected 10 A. Excretory acidosis hours after irradiation? B. Nongaseous alkalosis A. Neutrophilic leukocytosis C. Gaseous acidosis B. Agranulocytosis D. Metabolic acidosis C. Leukopenia E. Gaseous alkalosis D. Leukemia 168. After a 2 y.o. child has had flu, there E. Lymphocytosis appeared complaints about ear ache. A 163. In case of a penetrating wound of the doctor revealed hearing impairment and anterior abdominal wall the wound tract inflammation of the middle ear. How did went above the lesser curvature of stomach. the infection penetrate into the middle ear? What peritoneum formation is most likely A. Through the auditory tube to be injured? B. Through foramen jugu1aris A. Ligamentum triangulare sinistrum C. Through canalis caroticus B. Ligamentum hepatorenale D. Through canalis nasolacrimalis C. Ligamentum gastrocolicum E. Through atrium mastoideum D. Ligamentum hepatogastricum 169. In course of an experiment a skeletal E. Ligamentum hepatoduoduodenale muscle is being stimulated by a series of 164. An electron micrograph of a kidney electric impulses. What type of muscle fragment presents an afferent arteriole. contraction will arise, if every subsequent Under its endothelium some big cells can be impulse comes in the period of shortening seen that contain secretory granules. What of the previous single muscle contraction? type of cells is it? A. Holotetanus A. Juxtaglomerular B. Partial tetanus B. Smooth muscle cells C. A series of single contractions C. Interstitial D. Muscle contracture D. Juxtavascular E. Asynchronous tetanus E. Mesangeal 170. Examination of a miner revealed 165. Arterial pressure of a surgeon who pulmonary fibrosis accompanied by disturbance performed a long operation rised up to of alveolar ventilation. What is 140/110 mm Hg. What changes of humoral the main mechanism of this disturbance? regulation could have caused the rise of A. Limitation of respiratory surface of lungs arterial pressure in this case? B. Constriction of superior respiratory tracts A. Activation of renin angiotensive system C. Bronchi spasm B. Activation of sympathoadrenal system D. Disturbance of neural respiration control C. Inhibition of sympathoadrenal system E. Limitation of breast mobility D. Activation of formation and excretion of 171. A patient has symptoms of inflammation aldosterone of urogenital tracts. Examination of a E. Activation of kallikrein kinin system vaginal smear revealed big monocellular, 166. A patient with chronic cardiac insufficiency pear-shaped organisms with the pointed spike has been treated with cardiotonic drugs at the posterior end of body, big nucleus and a thiazide diuretic, but in spite of it there and undulating membrane. What protozoa are still edemata and risk of ascites. What were found in the smear? medication should be prescribed to amplify A. Trypanosoma gambiense diuretic effect of the applied drugs? B. Trichomonas vaginalis A. Spironolactone C. Trichomonas hominis B. Furosemide D. Trichomonas buccalis
  • 18. E. Lamblia interstinalis B. Verapamil 172. A child is languid, apathetic. Liver C. Nitroglycerin is enlarged and liver biopsy revealed a D. Metoprolol significant excess of glycogene. Glucose E. Propranolol concentration in the blood stream is below 177. A 63 y.o. man with collapse symptoms normal. What is the cause of low glucose was delivered to the emergency hospital. concentration? A doctor chose noradrenaline in order to A. Deficit of a gene that is responsible prevent hypotension. What is the action for synthesis of glucose 1-phosphaturidine mechanism of this medication? transferase A. Activation of dopamine receptors B. Low (absent) activity of hexokinase B. Block of M -cholinoreceptors C. High activity of glycogen synthetase C. Activation of serotonin receptors D. Low (absent) activity of glucose 6- D. Activation of α1-adrenoreceptors phosphatase E. Activation of β-adrenoreceptors E. Low (absent) activity of glycogene 178. Urine examination of a patient with phosphorylase in liver acute cystitis revealed leukocytes and 173. A man who went for a ride on a a lot of gram-negative bacilli. Inoculation roundabout had amplification of heart rate, resulted in growth of colonies of mucous sweating and nausea. What receptors stimulation nature that formed green soluble pigment. is it primarily connected with? What microorganism is the most probable A. Proprioceptors cause of the disease? B. Visual A. Proteus mirabilis C. Auditory B. Escherihia coli D. Tactors C. Salmonella enteritidis E. Vestibular D. Klebsiella pneumoniae 174. Two days after consumption of E. Pseudomonas aeruginosa smoked pork a patient got face and eyelid 179. An isolated cell of human heart edemata, gastrointestinal disturbances, automatically generates excitation impulses abrupt temperature rise, muscle pain. Blood with frequency 60 times pro minute. What analysis showed full-blown eosinophilia. heart structure was this cell obtained from? What helminth could the patient be infected A. Ventricle with? B. Atrium A. Trichina C. Sinoatrial node B. Hookworm D. Atrioventricular node C. Whipworm E. His' bundle D. Ascarid 180. A patient died under conditions of E. Pinworm cardiovascular insufficiency. Autopsy results: 175. Examination of a patient revealed postinfarction cardiosclcrosis, myocardium hyperkaliemia and hyponatremia. Low hypertrophy and dilatation of its cavities, secretion of which hormone may cause such especially of its right ventricle. Liver changes? is enlarged, its surface is smooth, incision A. Natriuretic revealed that it was plethoric, with dark-red B. Parathormone specks against the background of brownish C. Cortisol tissue. Histologically: plethora of central D. Aldosteron parts of lobules; peritheral parts around E. Vasopressin portal tracts contain hepatocytes in a state 176. After a tooth extraction a patient felt of adipose degeneration. How are these liver persistent pain behind his breast bone. After changes called? sublingual intake of an antianginal drug the A. Nutmeg liver pain behind the breast bone disappeared, B. Liver cirrhosis but the patient complained of headache and C. Liver steatosis dizziness. What drug are these properties D. Pseudonutmeg liver typical for? E. Amyloidosis A. Validol 181. Autopsy of a newborn boy revealed
  • 19. polydactylia, microcephalia, cheiloschisis D. Acute erythromyelosis and uranoschisis as well as hypertrophy E. Acute myeloblastic leukosis of parenchimatous organs. These defects 186. Continious taking of a drug can result correspond with the description of Patau's in osteoporosis. erosion of stomach mucous syndrome. What is the most probable cause membrane, hypokaliemia, retention of sodium of this pathology? and water, reduced content of corticotropin A. Trisomy of the 18th chromosome in blood. Name this drug: B. Trisomy of the 21st chromosome A. Digoxin C. Trisomy of the 13th chromosome B. Indometacin D. Nondisjunction of sex chromosomes C. Reserpine E. Partial monosomy D. Prednisolone 182. Glutamate decarboxylation results in E. Hydrochlorothiazide formation of inhibitory transmitter in CNS. 187. A patient has a haemorrhage into the Name it: posterior central gyrus. What type of sensitivity A. Serotonin on the opposite side will be disturbed? B. Histamine A. Visual C. Glutathione B. Auditory and visual D. Asparagine C. Auditory E. GABA D. Olfactory 183. A patient who suffers from acute E. Skin and proprioceptive myocarditis has clinical signs of cardiogenic 188. As a result of an accident a patient shock. What of the under-mentioned has intense painfullness and edema of the pathogenetic mechanisms plays the main anterior crus surface: dorsal f1exion of foot part in shock development? is hindered. Function of which crus muscle A. Disturbance of pumping ability of heart is most likely to be disturbed? B. Depositing of blood in organs A. M.peroneus longus C. Reduction of diastolic flow to the heart B. M.flexor digitorum longus D. Decrease of vascular tone C. M.flexor hallucis longus E. Increase of peripheral vascular resistance D. M.peroneus brevis 184. A patient who has been treated E. M.tibialis anterior with diazepam on account of neurosis 189. A 2 y.o. child has convulsions as a result complains of toothache. Doctor administered of lowered concentration of calcium ions in him an analgetic, but its dose was lower than blood plasma. It is caused by reduced function average therapeutic dose. What of: phenomenon did the doctor take into A. Adrenal cortex account while prescribing the patient an B. Pineal gland underdose? C. Hypophysis A. Summation D. Parathyroid glands B. Potentiation E. Thymus C. Cumulation 190. Histological specimen presents a vessel D. Tolerance the wall of which consists of endothelium, E. Drug dependence basal membrane and loose connective tissue. 185. A 23 y.o. patient complains of What type of vessel is it? weakness, temperature rise up to 38-40°C. A. Hemocapillary Objectively: liver and spleen are enlarged. B. Lymphocapillary Hemogram: Hb- 100 g/L erythrocytes - C. Vein of non-muscular type 2.9*1012/l, leukocvtes - 4.4· l09/l, thrombocytes D. Vein of muscular type - 48*109 /1, segmentonuclear neutrophils 17% E. Artery lymphocytes - 15%, blast cells - 68%. 191. Examination of a man who hadn't been All cytochemical reactions are negative. consuming fats but had been getting enough Make a hematological conclusion: carbohydrates and proteins for a long time A. Undifferentiated leukosis revealed dermatitis, poor wound healing, vision B. Acute lymphoblastic leukosis impairment. What is the probable cause C. Chronic myeloleukosis of metabolic disorder?
  • 20. A. Low caloric value of diet necrosis, fibrinous thrombs in their lumens; B. Lack of vitamins PP, H sclerosis and hyalinosis of glomerules, C. Lack of palmitic acid atrophy of tubules and fibrosis of renal D. Lack of linoleic acid, vitamins A, D, E, K stroma. What is the most probable diagnosis? E. Lack of oleic acid A. Membranous nephropathy 192. Analysis of a punction biopsy material B. Acute glomerulonephritis of liver revealed hepatocyte dystrophy with C. Focal segmentary sclerosis necroses as well as sclerosis with disorder of D. Subacute glomerulonephritis beam and lobulous structure, with formation E. Chronic glomerulonephritis of pseudolobules and regenerative nodes. 197. To prevent postoperative bleeding What is the most probable diagnosis: a 6 y.o. child was administered vicasol A. Acute hepatitis that is a synthetic analogue of vitamin K. B. Progressive massive liver necrosis Name post-translational changes of blood C. Liver cirrhosis coagulation factors that will be activated by D. Chronic hepatosis vicasol: E. Chronic hepatitis A. Phosphorylation of serine radicals 193. A 50 y.o. patient was admitted to the B. Polymerization hospital with complaints about pain behind C. Partial proteolysis his breastbone, asphyxia during physical D. Glycosyiation activity. Angiography revealed pathological E. Carboxylation of glutamin acid changes in the posterior interventricular 198. According to clinical indications a patient branch of the right eoronary artery. What was administered pyridoxal phosphate. heart parts are affected? What processes is this medication intended A. Posterior wall of the right and left ventricles to correct? B. Right atrioventricular valve A. Oxidative decarboxylation of ketonic C. Right atrium acids D. Left atrium B. Transamination and decarboxylation of E. Anterior wall of the right and left ventricles aminoacids 194. A hepatitis outbreak was registered in C. Desamination of purine nucleotide a settlement. This episode is connected with D. Synthesis of purine and pyrimidine bases water factor. What hepatitis virus could E. Protein synthesis have caused the infective outbreak in this 199. A 62 y.o. woman complains of settlement? frequent pains in the area of her chest and A.E backbone, rib fractures. A doctor assumed B.C myelomatosis (plasmocytoma). What of the C.B following laboratory characteristics will be D.D of the greatest diagnostical importance? E.G A. Hyperalbuminemia 195. Examination of coronary arteries B. Proteinuria revealed atherosclerotic calcific plaques that C. Hypoglobulinemia close vessel lumen by 1/3. The muscle has D. Paraproteinemia multiple whitish layers of connective tissue. E. Hypoproteinemia What process was revealed in myocardium? 200. A hypertensive glucose solution was A. Postinfarction cardiosclerosis introduced to a patient. It will intensify B. Tiger heart water movement: C. Myocardium infarction A. From the capillaries to the intercellular D. Myocarditis liquid E. Diffuse cardiosclerosis B. There will be no changes of water 196. Microscopical renal examination of a movement 36 y.o. woman who died from renal insufficiency C. From the intercellular liquid to the cells revealed in the glomerules proliferation D. From the cells to the intercellular liquid of capsule nephrothelium as well as E. From the intercellular liquid to the capillaries. of podocytes and phagocytes accompanied by formation of "crescents", capillary loop